Patho Test #3

Réussis tes devoirs et examens dès maintenant avec Quizwiz!

A 20-year-old has been admitted to a rehabilitation center after hospital treatment for an ischemic stroke. Which of the following aspects of the client's history would be considered to have contributed to his stroke? Select all that apply. The client: A) is an African American male. B) takes iron supplements for the treatment of chronic anemia. C) blood pressure has historically been in the range of 150s/90s. D) was diagnosed with type 2 diabetes 8 years ago. E) takes corticosteroids for the treatment of rheumatoid arthritis.

A, C, D African American race, male gender, hypertension, and diabetes are all well-documented risk factors for stroke. Anemia, autoimmune disorders like rheumatoid arthritis, and the use of corticosteroids are not noted to predispose to stroke.

A 55-year-old man has been diagnosed with a gastroesophageal reflux disease (GERD), in which the function of his lower esophageal sphincter is compromised. Which of the following consequences of this condition is most likely to occur? A) Decreased absorption of ingested foods and fluids B) Impaired control of the gastric emptying rate C) Protrusion of the stomach or regurgitation of stomach contents into the esophagus D) Inappropriate release of gastric enzymes

C Given that the role of the lower esophageal sphincter is to control the exchange of foods and fluids, a deficit is likely to allow the stomach contents, or the stomach itself, to protrude into the esophagus. Absorption is unlikely to be directly affected, and the sphincter is not responsible for controlling gastric emptying or enzyme secretion.

Which of the following individuals would most likely experience global ischemia to his or her brain? A) A male client who has just had an ischemic stroke confirmed by CT of his head B) A woman who has been admitted to the emergency department with a suspected intracranial bleed C) A man who has entered cardiogenic shock following a severe myocardial infarction D) A woman who is being brought to hospital by ambulance following suspected carbon monoxide poisoning related to a faulty portable heater

C Global ischemia is associated with a cessation of blood flow to the entire brain, as often occurs during cardiac arrest or myocardial infarction. Ischemic stroke and intracranial bleeding are likely to cause focal ischemia; carbon monoxide toxicity is associated with hypoxia.

A child is experiencing difficulty with chewing and swallowing. The nurse knows that which of the following cells may be innervating specialized gut-related receptors that provide taste and smell? A) Special somatic afferent fibers B) General somatic afferents C) Special visceral afferent cells D) General visceral afferent neurons

C Special visceral afferent cells innervate specialized gut-related receptors. Their central processes communicate with special VIA column neurons that project to reflex circuits producing salivation, chewing, swallowing, and other responses. Special somatic afferent fibers are concerned with joint and tendon sensation. General somatic afferents respond to stimuli that produce pressure or pain. General visceral afferent neurons innervate visceral structures such as the GI tract, urinary bladder, and the heart and great vessels.

A dialysis technician is providing care for a client with chronic renal failure. The technician would recognize which of the following characteristics of healthy kidneys? Select all that apply. A) The kidneys are contained within the peritoneal cavity. B) Blood vessels, nerves, and ureters all connect with the kidney at the hilus. C) The medulla of the kidney contains the glomeruli. D) Each kidney consists of lobes, with each lobe comprised of nephrons. E) Each nephron contains several hundred glomeruli that perform filtration.

B, D The hilus is the point of nerve and blood input and urine output for each kidney, and each kidney is composed of up to 18 lobes. The kidneys are outside the peritoneal cavity, and glomeruli exist in the outer cortex. Each nephron contains just one glomerulus.

When explaining a cystometry test to measure bladder pressure during filling and voiding in a normal adult, the nurse informs the nursing students that the normal capacity when adults have a desire to void is A) 100 to 150 mL. B) 200 to 250 mL. C) 300 to 399 mL. D) 400 to 500 mL.

D The desire to void occurs when the bladder is full (normal capacity is approximately 400 to 500 mL). At this point, a definite sensation of fullness occurs; the pressure rises sharply to 40 to 100 cm H2O; and voiding occurs around the catheter.

Which of the following clients' signs and symptoms would allow a clinician to be most justified in ruling out stroke as a cause? An adult A) has had a gradual onset of weakness, headache, and visual disturbances over the last 2 days. B) has experienced a sudden loss of balance and slurred speech. C) has vomited and complained of a severe headache. D) states that his left arm and leg are numb, and gait is consequently unsteady.

A A cardinal trait of the manifestations of stroke is that the onset is sudden, and a gradual onset of symptoms over 2 days would suggest an alternative etiology. Ataxia, slurred speech, and unilateral numbness are associated with stroke, with sudden vomiting and headache being particularly indicative of a hemorrhagic CVA.

Which of the following phenomena contributes to the difficulties with absorption, distribution, and elimination of drugs that are associated with kidney disease? A) Reductions in plasma proteins increase the amount of free drug and decrease the amount of protein-bound drug. B) Acute tubular necrosis is associated with impaired drug reabsorption through the tubular epithelium. C) Decreased retention by the kidneys often renders normal drug dosages ineffective. D) Dialysis removes active metabolites from circulation minimizing therapeutic effect.

A A decrease in plasma proteins, particularly albumin, that occurs in many persons with renal failure results in less protein-bound drug and greater amounts of free drug. Drug elimination problems do not stem as directly from impaired tubular reabsorption, decreased retention, or the process of dialysis.

Following a collision while mountain biking, the diagnostic workup of a 22-year-old male has indicated the presence of an acute subdural hematoma. Which of the following pathophysiological processes most likely underlies his diagnosis? A) Blood has accumulated between the man's dura and subarachnoid space. B) Vessels have burst between the client's skull and his dura. C) A traumatic lesion in the frontal or temporal lobe has resulted in increased ICP. D) Blood has displaced CSF in the ventricles as a consequence of his coup-contrecoup injury. Ans:

A A subdural hematoma develops in the area between the dura and the arachnoid space, while epidural hematomas exist between the skull and dura. Intracerebral hematomas are located most often in the frontal or temporal lobe, and the ventricles are not directly involved in a subdural hematoma.

To treat enuresis in a young girl, her pediatrician prescribes desmopressin, an antidiuretic hormone (ADH) nasal spray, before bedtime. What is the most likely rationale for this treatment? A) It removes water from the filtrate and returns it to the vascular compartment. B) It lessens the amount of fluid entering the glomerulus. C) It leads to the production of dilute urine. D) It causes tubular cells to lose their water permeability.

A ADH maintains extracellular volume by returning water to the vascular compartment. This leads to the production of concentrated urine by removing water from the tubular filtrate. In exerting its effect, ADH produces a marked increase in water permeability in tubular cells.

A 29-year-old female has been admitted to the emergency department following a suicide attempt by overdose of acetaminophen. What changes in the client's liver and diagnostic results would the care team most likely anticipate? A) Hepatocellular necrosis evidenced by increased ALT and AST levels B) Allergic inflammation accompanied by an increase in serum IgE and basophils C) Cholestatic reaction with increased bilirubin count D) Rapid onset of hepatitis and increased GGT, ALT, and bilirubin

A Acetaminophen is commonly implicated in cases of direct hepatotoxicity, a situation that is characterized by hepatocellular necrosis and increased ALT and AST levels. An allergic-type reaction is associated with idiosyncratic drug reactions, and cholestatic reactions and development of hepatitis are not noted to be associated with acetaminophen.

A 60-year-old man has long managed his type 1 diabetes effectively with a combination of vigilant blood sugar monitoring, subcutaneous insulin administration, and conscientious eating habits. This morning, however, his wife has noted that he appears pale and clammy and appears to be in a stupor, though he is responsive. She suspects that he has made an error in his insulin administration and that he is experiencing a hypoglycemic episode. Which of the following actions should be the wife's first choice? A) Administration of 15 to 20 g of glucose in a concentrated carbohydrate source B) Administration of subcutaneous glucagon C) IV infusion of 50% dextrose and water solution D) Careful monitoring for level of consciousness and resolution of hypoglycemia

A An insulin reaction necessitates intervention in addition to careful observation. The ideal response to an insulin reaction in a still, conscious client is the administration of 15 to 20 g of glucose in a concentrated carbohydrate source. Glucagon or D50 would be indicated if the client is unconscious or unable to swallow.

hich of the following clients in a hospital medical unit is most clearly demonstrating the signs and symptoms of liver failure? An adult with A) low hemoglobin levels, low platelet levels, and spider angiomas present. B) blood pressure of 189/103, jaundice, and multiple thromboses. C) sudden onset of confusion, a history of alcohol abuse, and low levels of serum AST and ALT. D) ascites, fever, and recent onset of atrial fibrillation.

A Anemia, thrombocytopenia, and the presence of spider angiomas are characteristic of liver failure. High blood pressure, excessive clotting, fever, and cardiac arrhythmias are not common symptoms of liver failure, and AST and ALT levels would rise, not fall.

The unique clinical presentation of a 3-month-old infant in the emergency department leads the care team to suspect botulism. Which of the following assessment questions posed to the parents is likely to be most useful in the differential diagnosis? A) "Have you ever given your child any honey or honey-containing products?" B) "Is there any family history of neuromuscular diseases?" C) "Has your baby ever been directly exposed to any chemical cleaning products?" D) "Is there any mold in your home that you know of?"

A Botulism in infants is frequently attributable to honey. Family history is not a relevant consideration given the bacterial etiology, and mold and chemical cleaning products are not known to predispose to botulism toxicity.

A badly burned firefighter has been in an induced coma for 3 weeks. When he awakens, he thanks his son for singing Happy Birthday to him a week earlier. Which part of the brain is responsible for allowing him to hear and comprehend while comatose? A) Thalamus B) Hypothalamus C) Corpus callosum D) Basal ganglia

A Coordination and integration of peripheral sensory stimuli occur in the thalamus, along with some crude interpretation of highly emotion-laden auditory experiences that not only occur but also can be remembered. For example, a person can recover from a deep coma in which cerebral cortex activity is minimal and remember some of what was said at the bedside. Inferior to the thalamus, and representing the ventral horn portion of the diencephalon, is the hypothalamus; it is the area of master-level integration of homeostatic control of the body's internal environment. The corpus callosum is a massive commissure, or bridge, of myelinated axons that connects the cerebral cortex of the two sides of the brain. The basal ganglia lie on either side of the internal capsule, just lateral to the thalamus; they supply axial and proximal unlearned and learned postures and movements, which enhance and add gracefulness to UMN-controlled manipulative movements.

A 28-year-old man presents with complaints of diarrhea, fecal urgency, and weight loss. His stool is light colored and malodorous, and it tends to float and be difficult to flush. He has also noted tender, red bumps on his shins and complains of pain and stiffness in his elbows and knees. Sigmoidoscopy reveals discontinuous, granulomatous lesions; no blood is detected in his stool. Which of the following diagnoses would his care team first suspect? A) Crohn disease B) Ulcerative colitis C) Diverticulitis D) Colon cancer

A Crohn disease, like ulcerative colitis, causes diarrhea, fecal urgency, weight loss, and systemic symptoms such as erythema nodosum and arthritis. Unlike ulcerative colitis, it also causes steatorrhea but is not as likely to cause blood in the stool. The granulomatous "skip" lesions confirm the diagnosis of Crohn disease. Neither diverticulitis nor colon cancer would cause this combination of symptoms and signs.

During a clinical assessment of a 68-year-old client who has suffered a head injury, a neurologist suspects that a client has a sustained damage to her vagus (CN X) nerve. Which of the following assessment findings is most likely to lead the physician to this conclusion? A) The client has difficulty swallowing and has had recent constipation and hypoactive bowel sounds. B) The client is unable to turn her head from side to side, and her tongue is flaccid. C) The client has a unilateral facial droop, dry eyes, and decreased salivary production. D) The client is unable to perform any fine motor movements of her tongue.

A Dysphagia and impaired GI motility are associated with damage to the vagus nerve. Lateral movement of the head is mediated by CN XI. Facial droop and dry eyes are associated with CN VII, the facial nerve, while abnormal tongue movement is a result of damage to CN XII, the hypoglossal nerve.

A 68-year-old woman with a new onset of vascular dementia has recently begun retaining urine. Which of the following physiological phenomena would her care providers most realistically expect to currently occur as a result of her urinary retention? A) Hypertrophy of the bladder muscle and increased bladder wall thickness B) Decreased urine production and nitrogenous waste excretion by the kidneys C) Decompensation, bladder stretching, and high residual urine volume D) Overflow incontinence and loss of contraction power

A Early accompaniments to urinary retention include hypertrophy of the bladder muscle and increased thickness of the bladder wall. Renal effects are unlikely, and decompensation and loss of contraction power are most often later rather than early effects.

When educating a patient about to undergo a pacemaker insertion, the nurse explains the normal phases of cardiac muscle tissue. During the repolarization phase, the nurse will stress that membranes must be repolarized before they can be reexcited. Within the cell, the nurse understands that A) potassium channels open while sodium channels close, causing repolarization to the resting state. B) the influx of calcium is the primary stimulus for the repolarization of cardiac tissue. C) only the electrical activity within the heart will determine when repolarization occurs. D) the cell membranes need to stay calm resulting in muscle tissue becoming refractive.

A Repolarization is the phase during which the polarity of the resting membrane potential is reestablished. This occurs with the closure of the sodium channels and opening of the potassium channels.

A number of clients on a geriatric subacute medical floor of a hospital have developed foul-smelling diarrhea over the last several days, and subsequent culture of stool samples has confirmed the presence of Clostridium difficile in each case. The care team in the unit would recognize that which of the following factors likely contributed to the health problem and would anticipate which of the following treatments? A) The use of broad-spectrum antibiotics likely played a role in the development of infections, and most clients would likely receive metronidazole as a treatment. B) Genetic predisposition and the presence of the bacterium in clients' normal flora likely contributed, and treatment would consist of broad-spectrum antibiotics. C) Poor hand washing practice on the part of care providers led to the outbreak, and treatment will consist of hydration and nutritional support. D) Ingestion of contaminated food probably contributed to the infections, and corticosteroids will be needed to treat them.

A Elimination of the normal intestinal flora by broad-spectrum antibiotics commonly precedes infection by C. difficile, and metronidazole is the normal treatment. Neither genetic predisposition nor ingestion of contaminated food is a likely factor. While poor hand hygiene can spread the bacteria, treatment for C. difficile necessitates antibiotics and not simply hydration and nutritional support. The treatment of C. difficile is with antibiotics. Metronidazole is the drug of choice with vancomycin, being reserved for people who cannot tolerate metronidazole. Corticosteroids will not kill the bacterium.

A woman with a diagnosis of type 2 diabetes has been ordered a hemoglobin A1C test for the first time by her primary care provider. The woman states, "I don't see why you want to test my blood cells when its sugar that's the problem." What aspect of physiology will underlie the care provider's response to the client? A) The amount of glucose attached to A1C cells reflects the average blood glucose levels over the life of the cell. B) Hemoglobin synthesis by the bone marrow is inversely proportionate to blood glucose levels, with low A1C indicating hyperglycemia. C) The high metabolic needs of red cells and their affinity for free glucose indicate the amount of glucose that has been available over 6 to 12 weeks. D) Insulin is a glucose receptor agonist on the hemoglobin molecule, and high glucose suggests low insulin levels.

A Glucose entry into red blood cells is not insulin dependent, and the rate at which glucose becomes attached to the hemoglobin molecule depends on blood glucose levels. A1C levels thus indirectly indicate glucose levels. Hemoglobin synthesis, the metabolic needs of hemoglobin, and an agonist role of insulin do not underlie the A1C test.

During a flu shot clinic, one of the questions the student nurse asks relates to whether the patient has had Guillain-Barré syndrome in his medical history. The patient asks, "What is that?" How should the nursing student reply? A) "A type of paralysis that affects movement on both sides of the body that may even involve the respiratory muscles" B) "Swelling of your arm where you got your flu shot, and maybe your eyes and lips had some swelling as well" C) "A degenerative disease where you have trouble walking without the help of a cane or walker" D) "Influenza-like illness where you had fever and chills for 2 to 3 days after your last flu shot"

A Guillain-Barré syndrome is an acute immune-mediated polyneuropathy. The majority of people report having had an acute, influenza-like illness before the onset of symptoms. It progresses along the ascending muscle weakness of the limbs, producing a symmetric flaccid paralysis. The rate of disease progression varies, and there may be disproportionate involvement of the upper or lower extremities. Option B is anaphylaxis following the flu shot. It is not a degenerative disease.

A pancreatitis patient is admitted with weight loss, nausea, and vomiting. To maintain nutrition, the physician orders parental nutrition to be started. Knowing that a major side effect of parenteral nutrition is a hyperosmolar hyperglycemic state, the nurse should assess the patient for which clinical manifestation? A) Dry lips, excess urine output, and seizures B) Facial tics, shuffling gait, and stiff joints C) Fever, chills, and elevated BP of 170/101 D) Irritability, bradycardia, and wheezing noted on inspiration

A Hyperosmolar hyperglycemic state is characterized by high blood glucose (>600 mg/dL), dehydration (dry lips), depression of sensorium, hemiparesis, seizures, and coma, and also weakness, polyuria, and excessive thirst. HHS may occur in various conditions, including type 2 diabetes, acute pancreatitis, severe infection, MI, and treatment with oral or parenteral nutrition solutions.

A 73-year-old man presents to his family physician with complaints of recent urinary hesitation and is eventually diagnosed with benign prostatic hyperplasia (BPH). Which of the following clinical consequences would his care provider expect prior to the resolution of his health problem? A) Hydroureter and pain B) Development of renal calculi and renal cysts C) Unilateral hydronephrosis and pain D) Development of glomerulonephritis or nephrotic syndrome

A Pain and the distention of the distal ureter would be expected manifestations of BPH. Renal calculi, cysts, glomerulonephritis, and nephrotic syndrome are unlikely to develop consequentially, and unilateral hydronephrosis is unlikely, given that the obstruction is below the level of the ureterovesical junction.

A stroke patient is having difficulty swallowing food and beverages. The patient complains that he feels like "the food is sticking to the back of his throat." Given this information, the priority nursing interventions would be to A) make the patient "nothing per os" (NPO) and call the physician. B) feed the patient while he is sitting in an upright position. C) add a thickening agent to all of the patient's beverages. D) warrant no action since this is a normal occurrence after a stroke.

A People with dysphagia usually complain of choking, coughing, or an abnormal sensation of food sticking in the back of the throat or upper chest when they swallow. A neuromuscular cause involves lesions of the CNS, such as a stroke, which often involve the cranial nerves that control swallowing. Feeding in upright position is good once it is determined by swallowing evaluation that the patient can swallow food without it going into the lungs. Likewise, thickening agents help dysphagia patients after a swallow evaluation has been performed. No action could put the patient at risk for aspiration pneumonia.

A 1-year-old baby boy with renal dysplasia risks end-stage renal disease unless intervention occurs. Which of the following treatment options is his care team most likely to reject? A) Dietary restriction plus erythropoietin B) Continuous cyclic peritoneal dialysis C) Renal transplantation D) Continuous ambulatory peritoneal dialysis

A Renal transplantation and dialysis are recommended for children; of these, transplantation is the preferred treatment. Conservative measures are inappropriate in this age group because of the importance of fostering proper bone growth, especially in the first 2 years, and appropriate cognitive development, which is at risk due to issues such as uremic encephalopathy and the effect of renal failure upon the central nervous system of the developing child.

A hospital client with a diagnosis of chronic renal failure has orders for measurement of her serum electrolyte levels three times per week. Which of the following statements best captures the relationship between renal failure and sodium regulation? A) Clients with advanced renal failure are prone to hyponatremia because of impaired tubular reabsorption. B) Renal clients often require a sodium-restricted diet to minimize the excretion load on remaining nephrons. C) Clients with renal failure often maintain high sodium levels because of decreased excretion. D) Restricting sodium intake helps to preserve nephron function and has the additional benefit of lowering blood pressure.

A The compromised ability of the tubular nephrons to reabsorb sodium predisposes renal clients to low serum sodium levels. A sodium restriction is thus not normally indicated.

Which of the following clients on a medical unit of a hospital is most likely to be experiencing health problems that may be attributable to kidney disease? A) An 81-year-old female client with osteoporosis and anemia B) A 77-year-old client with urinary retention due to benign prostatic hyperplasia (BPH) C) A 55-year-old woman with a recent stroke secondary to long-standing hypertension D) A 60-year-old man with a systemic fungal infection requiring intravenous antibiotics.

A The kidneys play central roles in both vitamin D activation and regulation of red blood cell production, deficiencies of which may result in osteoporosis and anemia, respectively. BPH, stroke, and infection are less likely to result from kidney disease.

A 45-year-old diabetic male is experiencing erectile dysfunction. If his erectile dysfunction is caused by the nervous system, then the nurse can educate the patient that the venous blood supply to the penis is controlled by A) sacral parasympathetic fibers. B) the hypothalamus. C) the vagus nerve. D) postganglionic sympathetic neurons.

A The pelvic nerves leave the sacral plexus on each side of the cord and distribute their peripheral fibers to the bladder, uterus, urethra, prostate, distal portion of the transverse colon, descending colon, and rectum. Sacral parasympathetic fibers also supply the venous outflow from the external genitalia to facilitate erectile function. The hypothalamus, vagus nerve, and postganglionic sympathetic neurons do not control erectile function.

An 87-year-old male resident of an assisted living facility has been consistently continent of urine until the last several weeks. Which of the following actions by the care providers at the facility is the most likely priority? A) Performing a physical examination and history to determine the exact cause and character of the incontinence B) Providing client education focusing on the fact that occasional incontinence is a normal, age-related change C) Teaching the resident about protective pads, collection devices, and medications that may be effective D) Showing the resident the correct technique for exercises to improve bladder, sphincter, and pelvic floor tone

A The priority in the treatment of incontinence in the elderly is an acknowledgement that it is not an inevitability and that the exact causes should and most often can be identified. This identification by way of history-taking and examination would supersede teaching about protective devices or exercises.

A male patient has just been diagnosed with esophageal cancer. He knew that he was losing weight and fatigued most days, but he just attributed it to aging and working. The physician recommends chemotherapy and irradiation. However, the cancer has already metastasized. The patient asks the nurse what he can expect if he agrees to the treatments. The nurse responds, A) "The therapies may shrink the cancer." B) "The doctor is prescribing treatment measures to help you swallow better." C) "These therapies will most likely cure your cancer." D) "You need to talk with your physician some more. I will page him for you."

A The prognosis for people with cancer of the esophagus, although poor, has improved. Even with modern forms of therapy, the long-term survival is limited because, in many cases, the disease has already metastasized by the time the diagnosis is made. These therapies may help with food consumption, but that is not their primary purpose. Nor, will these therapies cure the cancer.

A 20-year-old has been diagnosed with an astrocytic brain tumor located in the brain stem. Which of the following statements by the oncologist treating the client is most accurate? A) "Your prognosis will depend on whether we can surgically resect your tumor." B) "Our treatment plan will depend on whether your tumor is malignant or benign." C) "This is likely a result of a combination of heredity and lifestyle." D) "The major risk that you face is metastases to your lungs, liver, or bones."

A The prognosis of people with pilocytic astrocytomas is influenced primarily by their location. The prognosis is usually better for people with surgically resectable tumors, such as those located in the cerebellar cortex, than for people with less accessible tumors, such as those involving the hypothalamus or brain stem. Because of infiltration of brain tissue that prevents total resection, surgery rarely cures brain tumors. The binary of malignant and benign is not used to characterize brain tumors, and the etiology and substantive risk factors are largely unknown. Brain tumors rarely metastasize outside the CNS.

A 3-year-old girl has just been diagnosed with type 1A diabetes. Her parents are currently receiving education from the diabetes education nurse. How can the nurse best explain to the parents the etiology (cause) of their daughter's diabetes? A) "The problem that underlies her diabetes is that her own body has destroyed the cells in her pancreas that produce insulin." B) "It's not known exactly why your daughter has completely stopped making insulin, and treatment will consist of your rigidly controlling her diet." C) "This tendency to produce insufficient amounts of insulin is likely something that she inherited." D) "Environmental and lifestyle factors are known to play a part in the fact that her pancreas secretes and withholds insulin at the wrong times."

A Type 1A, or immune-mediated, diabetes involves the autoimmune destruction of pancreatic beta cells and a consequent absolute lack of insulin. Exogenous insulin required as dietary control alone is insufficient. The central problem is an absolute lack of insulin production rather than deranged release.

Following the diagnosis of acute renal failure, the nurse knows that one of the earliest manifestations of residual tubular damage is which of the following lab/diagnostic results? A) Elevated blood urea nitrogen (BUN) B) Serum creatinine elevation C) Inability to concentrate urine D) Reduced glomerular filtration rate

A Urine tests that measure urine osmolality, urinary sodium concentration, and fractional excretion of sodium help differentiate prerenal azotemia, in which the reabsorptive capacity of the tubular cells is maintained, from tubular necrosis, in which these functions are lost. One of the earliest manifestations of tubular damage is the inability to concentrate urine. Conventional markers of serum creatinine and urea nitrogen, fractional secretion of sodium to assess glomerular filtration rate (GFR), and urine output do not manifest for 1 to 2 days after the acute renal failure has begun.

Which of the following substances is most likely to be reabsorbed in the tubular segments of the nephron using passive transport mechanisms? A) Water B) Sodium C) Phosphate D) Calcium

A Water is passively reabsorbed across tubular epithelial membranes, while ions like sodium, phosphate, and calcium necessitate active transport.

Which of the following diagnostic findings is likely to result in the most serious brain insult? A) Mean arterial pressure (MAP) that equals intracranial pressure (ICP) B) Moderate decrease in brain tissue volume secondary to a brain tumor removal C) Increased ICP accompanied by hyperventilation D) High intracellular concentration of glutamate

A When the pressure in the cranial cavity approaches or exceeds the MAP, tissue perfusion becomes inadequate; cellular hypoxia results; and neuronal death may occur. Displacement of CSF and blood can partially compensate for decreased brain tissue volume. Hyperventilation partially mitigates, rather than exacerbates, increase in ICP. Glutamate is normally in far higher concentrations intracellularly than extracellularly.

Parents have brought their 7-year-old child into the emergency room with abdominal pain. Which of the following clinical manifestations would lead the health care team to suspect the child has appendicitis? Select all that apply. A) Tenderness in right lower quadrant with palpation B) Rebound tenderness in inguinal areas with palpation C) Redness and warmth over right lower quadrant D) Bloating and flatulence noticeable E) Urine has the smell of stool with brown coloring

A, B Appendicitis usually has an abrupt onset. Palpation of the abdomen usually reveals a deep tenderness in the LRQ, which is confined to a small area approximately the size of the fingertip. It usually is located at approximately the site of the inflamed appendix. Rebound tenderness, which is pain that occurs when pressure is applied to the area and then released, and spasm of the overlying abdominal muscles are common. An abscess may display redness and warmth. Bloating and flatulence are usually seen with diverticular disease as is the development of fistula (vesicosigmoid), where stool is seen in the urine.

During a lecture about medication therapy for type 2 diabetic patients, the pharmacology instructor discusses side effects to monitor following subcutaneous injection of exenatide (Byetta), a glucagon-like peptide-1 (GLP-1) agonist. Of the following list, which signs and symptoms should the students be assessing in this patient? Select all that apply. A) Nausea B) Weight loss C) Dizziness D) Tachycardia E) Dehydration

A, B Exenatide is approved as an injectable monotherapy or combination adjunctive therapy for people with type 2 diabetes in association with diet and exercise. It is injected subcutaneously 60 minutes before a meal. The major side effects are nausea and weight loss. Dizziness and tachycardia are not listed as major side effects; however, they may be indicative of hypoglycemia related to other medications.

The nurse knows that a diabetic patient admitted for a lower limb infection likely is experiencing which of the following pathophysiologic principles listed below? Select all that apply. A) Many diabetic patients have sensory deficits and ignore minor trauma just because they can't feel it in their feet. B) Patients with chronic diabetes may have vascular problems that impair circulation, and therefore cells needed for adequate inflammatory response cannot reach the site. C) Hyperglycemia may provide an environment that enhances the growth of microorganisms. D) Infections happen to all age groups and people, and the diabetic patient has the same risk for developing an infection as other patients without diabetes. E) Any exercise that creates diaphoresis in the feet can cause a fungal infection in diabetic patients.

A, B, C Suboptimal response to infection in a person with diabetes is caused by the presence of chronic complications, such as vascular disease and neuropathies, and by the presence of hyperglycemia and altered neutrophil function. Certain types of infections occur with increased frequency in people with diabetes: soft tissue infections of the extremities, osteomyelitis, UTIs and pyelonephritis, candidal infections of the skin and mucous surfaces, dental caries, and periodontal disease.

While living and hiking in the Rocky Mountains, a gentleman slipped and fell. He goes to an urgent care where an x-ray was done, and some blood was drawn for a CBC. The clinic informs him that he is anemic. What may contribute to this person's anemia? Select all that apply. A) Living in a high altitude B) Tissue hypoxia C) Inability to manufacture erythropoietin D) Destruction of RBCs caused by natural killer cells E) Dehydration

A, B, C The synthesis of erythropoietin is stimulated by tissue hypoxia, which may be brought about by anemia, residing at high altitudes, or impaired oxygenation of tissues due to cardiac or pulmonary disease. Dehydration will cause an elevated level of RBCs.

Which of the following clinical manifestations would lead the nurse to suspect the renal failure patient is developing uremia? Select all that apply. A) Weakness and fatigue B) Lethargy and confusion C) Extreme itching D) Blood in urine E) Urine smell in the stool

A, B, C Uremia affects all body systems. The symptoms at the onset include weakness, fatigue, nausea, and apathy. These are subtle signs. More severe symptoms include extreme weakness, frequent vomiting, lethargy, and confusion.

A woman with poorly controlled type 1 diabetes has been admitted to a hospital unit for the treatment of ketoacidosis. Place the following events in the pathophysiology of ketoacidosis in the correct chronological order. Use all the options. A) Decrease in pH B) Breakdown of triglycerides C) Low serum insulin levels D) Ketone production by the liver E) Production of fatty acids and glycerol

A, B, C, D, E During ketoacidotic episodes, the lack of insulin leads to mobilization of fatty acids from adipose tissue because of the unsuppressed adipose cell lipase activity that breaks down triglycerides into fatty acids and glycerol. The increase in fatty acid levels leads to ketone production by the liver and ultimately metabolic acidosis.

A patient has just been admitted to a nursing unit with the diagnosis of obstructive jaundice. Which of the following assessment findings would the nurse expect to see in this patient? Select all that apply. A) Clay-colored stools B) Dark urine C) Elevated conjugated bilirubin levels D) Decreased serum alkaline phosphatase levels E) Severe itching

A, B, C, E Obstructive jaundice occurs when bile flow is obstructed between the liver and the intestine. Among the causes are strictures of the bile duct, gallstones, and tumors of the bile duct or the pancreas. Conjugated bilirubin levels usually are elevated; the stools are clay colored; the urine is dark; the levels of serum alkaline phosphatase are markedly elevated; and the aminotransferase levels are slightly increased. The accumulation of bile acids in the blood leads to the development of pruritus (itching).

A 41-year-old woman was diagnosed with multiple sclerosis (MS) 7 years ago and is sharing her story with members of an MS support group, many of whom have been diagnosed recently. Which of the following aspects of her health problem should the woman warn others to expect at some point in the progression of the disease? Select all that apply. A) Debilitating fatigue B) Progressive loss of visual acuity C) Gradual development of a resting tremor D) Loss of mental acuity E) Shuffling gait

A, B, D Fatigue, visual deficits, and cognitive disturbances are all noted manifestations of MS, while respiratory function and GI function are not normally affected. Shuffling gait is usually associated with Parkinson disease.

For several years, a 39-year-old female has been averaging two to three bottles of wine each night after her children go to sleep and has included several ounces of brandy in recent years as well. Despite negative consequences to her career and the dissolution of her marriage, her drinking has culminated in a diagnosis of cirrhosis. Which of the following physical manifestations of the health problem would her care team anticipate? Select all that apply. A) Ascites B) Anorexia C) Fever D) Bleeding tendencies E) Epigastric pain F) Obesity

A, B, D, E Ascites, anorexia, bleeding tendencies due to neurological effects, and epigastric pain are common accompaniments to cirrhosis. Ascites occurs when the amount of fluid in the peritoneal cavity is increased and is a late-stage manifestation of cirrhosis and portal hypertension. Epigastric pain is caused by liver enlargement or peritonitis. The peritoneal fluid is seeded with bacteria from the blood or lymph or from passage of bacteria through the bowel wall. Because factors V, VII, IX, and X, prothrombin, and fibrinogen are synthesized by the liver, their decline in liver disease contributes to bleeding tendencies. Fever and obesity would be less likely to exist, as the pathology is not infectious in nature and malnutrition and impaired food metabolism are common.

A female client with suspected glomerular disease has been referred to a nephrologist. The nurse knows that which of the following clinical manifestations may be present with the diagnosis of acute nephritic syndrome? Select all that apply. A) Sudden onset of hematuria B) Proteinuria C) Flank pain D) Excess urine output E) Edema

A, B, E In its most dramatic form, the acute nephritic syndrome is characterized by sudden onset of hematuria, variable degrees of proteinuria, diminished GFR, oliguria, and signs of impaired renal function. Inflammatory processes damage the capillary wall. This damage to the capillary wall allows RBCs to escape into the urine and produce a decrease in GFR. Extracellular fluid accumulation, hypertension, and edema develop because of the decreased GFR. Flank pain is usually associated with kidney stones. Oliguria occurs rather than excess urine output.

A brain tumor causing clinical manifestations of headache, nausea, projectile vomiting, and mental changes is likely located in which parts of the brain? Select all that apply. A) Intra-axially B) Extra-axially C) Brain stem D) Temporal lobe E) Frontal lobe

A, B, E Tumors within the intracranial (intra-axially) cavity are fixed and cause s/s of increased ICP like headache, nausea, vomiting, mental changes, papilledema, visual disturbances, and alterations in sensory and motor function. Outside the brain tissue (extra-axially), but within the cranium, tumors may reach large sizes without producing s/s. After they reach a sufficient size, s/s of increased ICP appear. Temporal lobe tumors often produce seizures as their first symptom. Brain stem tumors commonly produce upper/lower motor neuron s/s such as weakness of facial muscles and ocular palsies. Frontal lobe tumors also grow to a large size and cause s/s of increased ICP.

If the nurse suspects a spinal cord injury, the patient has developed autonomic dysreflexia. Which of the following assessments would confirm this complication? Select all that apply. A) BP 180/98 B) Skin covered with macular rash C) Pulse rate 49 D) Complains of a pounding headache E) Cold, cyanotic lower legs

A, C, D Autonomic dysreflexia represents an acute episode of exaggerated sympathetic reflex responses that occur in people with injuries at T6 and above, in which CNS control of spinal reflexes is lost. It is characterized by hypertension (BP 180/98), skin pallor, vagal slowing of the heart rate (pulse 49), and headache ranging from dull to severe and pounding.

A 54-year-old male who lives alone and has poorly controlled type 1 diabetes has been admitted to hospital for treatment of sepsis resulting from a chronic foot ulcer. Which of the other findings from the man's admission assessment and history would his care team likely to attribute to his diabetes? Select all that apply. A) The client complains of bloating after eating a meal and states that he is unable to eat much food at any one sitting. B) The man's resting heart rate is noted to be irregular. C) He states that his bowel movements are almost always loose. D) The client states that he has been unable to maintain an erection in recent years. E) Crackles are audible in his lower lung fields bilaterally. F) The man's temperature is 38.2°C (100.8°F) orally.

A, C, D Gastroparesis, diarrhea, and erectile dysfunction are frequent manifestations of diabetes, while cardiac dysrhythmias, pulmonary edema, and fever are not.

Which of the following clients would be considered to have a significant risk of developing the prerenal form of acute renal failure? Select all that apply. A) A 22-year-old male who has lost large amounts of blood following a workplace injury B) A 41-year-old female who is admitted for intravenous antibiotic treatment of pyelonephritis C) A 79-year-old male with diagnoses of poorly controlled diabetes mellitus and heart failure D) A 20-year-old male who is admitted for treatment of an overdose of a nephrotoxic drug E) A 68-year-old male with a diagnosis of benign prostatic hyperplasia (BPH) F) An 80-year-old female who has been admitted for the treatment of dehydration and malnutrition

A, C, F Hemorrhage, heart failure, and dehydration (hypovolemia) are all noted contributors to prerenal failure. Pyelonephritis and damage from nephrotoxic drugs would more likely result in intrinsic renal failure, while BPH is postrenal in nature.

A client has been diagnosed with having calcium oxalate kidney stones following intravenous pyelography. Which of the following teaching points about the treatment of the health problem are justifiable? Select all that apply. A) "You may need to cut out cocoa, chocolate, and some nuts from your diet." B) "It's important that you avoid high-calcium foods like milk, cheese, and yogurt." C) "We will come up with a plan to safely limit your fluid intake over the next few weeks." D) "Extracorporeal shock-wave lithotripsy treatment may be used to fragment larger stones." E) "Most likely your stones can be dissolved by medications over the next several days."

A, D Individuals with calcium oxalate stones often need to avoid high-oxalate foods like nuts, cocoa, and chocolate. Extracorporeal shock-wave lithotripsy treatment may be used to fragment larger renal calculi. It would not be necessary to avoid calcium intake, and fluid intake should encouraged, not curbed. Medications can reduce the potential for stone formation but are not a common treatment modality.

While explaining to a group of nursing students what the function of the first mucosal layer of the lower two thirds of the esophagus, the pathophysiology instructor mentions which of the following functions? Select all that apply. A) Secretion of mucus to lubricate and protect the inner surface of the alimentary canal B) Smooth muscle cells that facilitate movement of contents of the GI tract C) Holding the organs in place and storage of fats D) Barrier to prevent the entry of pathogenic organisms E) A cushioning to protect against injury from sports or car accidents

A, D The first layer performs numerous functions. These include production of mucus that lubricates and protects the inner surface of the alimentary canal; secretion of digestive enzymes and substances that break down food; absorption of the breakdown products of digestion; and maintenance of a barrier to prevent the entry of noxious substances and pathogenic organism. The facilitation of movement of contents of the GI tract occurs in the third layer while holding the organs in place, and storage of fats occurs in the fourth layer.

Because they strengthen the pelvic floor muscles, Kegel exercises are most likely to help A) overflow incontinence. B) urge incontinence. C) stress incontinence. D) mixed incontinence.

C Stress incontinence is commonly caused by weak pelvic floor muscles, which allow the angle between the bladder and the posterior proximal urethra to change so that the bladder and urethra are positioned for voiding when some activity increases intra-abdominal pressure. Overflow incontinence results when the bladder becomes distended and detrusor activity is absent. Urge incontinence is probably related to CNS control of bladder sensation and emptying or to the smooth muscle of the bladder. Mixed incontinence, a combination of stress and urge incontinence, probably has more than one cause.

A 22-year-old female with a history of intermittent flank pain, repeated UTIs, and hematuria has been diagnosed with autosomal dominant polycystic kidney disease (ADPKD). Which of the following phenomena has most likely contributed to the development of this diagnosis? A) UTIs coupled with an impaired immune response have caused her ADPKD. B) She has inherited a tendency for epithelial cells in her tubules to proliferate inappropriately. C) Severe hypertension and portal hypertension are likely precursors. D) She has inherited undersized kidneys that are prone to calculi formation.

B ADPKD is an inherited condition, and the etiology is thought to involve cysts arising in segments of the renal tubules from a few epithelial cells that proliferate abnormally. UTIs are consequent, not causative, of the condition. Severe hypertension and portal hypertension are more commonly associated with ARPKD than ADPKD. Kidneys are typically oversized in ADPKD, and renal calculi are not noted sequelae.

Which of the following patients on a geriatric medical unit is most likely to require slow-release potassium supplements on a regular basis? A) A 90-year-old female who is taking an aldosterone antagonist to treat pulmonary edema B) An 81-year-old male who takes a thiazide diuretic to control his hypertension C) A 79-year-old male with heart failure who is receiving a loop diuretic D) An 83-year-old female who is taking an osmotic diuretic to address severe peripheral edema

B A common side effect of thiazide diuretics is increased potassium losses in the urine, which may necessitate potassium supplementation. Aldosterone antagonists, loop diuretics, and osmotic diuretics are less likely to induce hypokalemia.

Following a motor vehicle accident 3 months prior, a 20-year-old female who has been in a coma since her accident has now had her condition declared a persistent vegetative state. How can her care providers most accurately explain an aspect of her situation to her parents? A) "Your daughter has lost all her cognitive functions as well as all her basic reflexes." B) "Though she still goes through a cycle of sleeping and waking, her condition is unlikely to change." C) "If you or the care team notices any spontaneous eye opening, then we will change our treatment plan." D) "Your daughter's condition is an unfortunate combination with total loss of consciousness but continuation of all other normal brain functions."

B A continuation of the sleep-wake cycle can exist in a persistent vegetative state. Reflexes often remain, as does spontaneous eye opening. Aspects of brain function beyond those governing consciousness are affected.

Which of the following clinical findings among older adults is most unlikely to warrant further investigation and possible treatment? A) An 81-year-old male's serum creatinine level has increased sharply since his last blood work. B) A 78-year-old female's GFR has been steadily declining over several years. C) A 90-year-old female's blood urea nitrogen (BUN) is rising. D) An 80-year-old male whose urine dipstick reveals protein is present.

B A gradual decrease in GFR is considered a normal age-related change. Increased creatinine or BUN would warrant follow-up, as would the presence of protein in a client's urine.

A patient has been admitted with diabetic ketoacidosis. The emergency department starts an IV to improve circulatory volume. If there is a sudden change in extracellular fluid osmolality that results in a too rapid blood glucose lowering, the nurse will likely observe which of the following clinical manifestations? A) Chills, profuse sweating, weakness B) Headaches, dizziness, change in level of consciousness C) Increase urine output, thirst, vomiting D) Rapid, deep respirations, palpitations, low BP

B A sudden change in the osmolality of extracellular fluid can also occur when blood glucose levels are lowered too rapidly, and this can cause cerebral edema, more common in children than in adults. Answers A, B, and C all have some component of signs and symptoms of DKA rather than cerebral edema signs and symptoms (answer B).

A 26-year-old female is resting after a one-minute episode during which she lost consciousness while her muscles contracted and extremities extended. This was followed by rhythmic contraction and relaxation of her extremities. On regaining consciousness, she found herself to have been incontinent of urine. What has the woman most likely experienced? A) A myoclonic seizure B) A tonic-clonic seizure C) An absence seizure D) A complex partial seizure

B A tonic-clonic seizure often begins with tonic contraction of the muscles with extension of the extremities and immediate loss of consciousness. Incontinence of the bladder and bowel is common. Cyanosis may occur from contraction of airway and respiratory muscles. The tonic phase is followed by the clonic phase, which involves rhythmic bilateral contraction and relaxation of the extremities. A myoclonic seizure involves bilateral jerking of muscles, generalized or confined to the face, trunk, or one or more extremities. Absence seizures are nonconvulsive, and complex partial seizures are accompanied by automatisms.

At which of the following locations in the nephron would a health care professional first expect blood to be largely free of plasma proteins? A) Proximal convoluted tubule B) Bowman space C) Loop of Henle D) Afferent arteriole

B Active filtration occurs when the whole blood enters via the afferent arteriole in to the glomerular capillaries, and the blood is then filtered into the Bowman space, removing plasma proteins. The filtrate that enters the proximal convoluted tubule and the loop of Henle is already free of plasma proteins.

A 43-year-old male client has presented to the emergency department with vomiting that he claims is of a sudden onset. The client also states that the emesis has often contained frank blood in the hours prior to admission. His vital signs are stable with temperature 98.3°F, pulse 88, BP 140/87, and respiratory rate 18. Which of the following potential contributing factors would the health care team suspect first? A) Overuse of antacids B) Alcohol consumption C) Staphylococcal enterotoxins D) Effects of Helicobacter pylori

B Acute gastritis associated with alcohol use is characterized by intermittent vomiting and the possibility of hematemesis. Aspirin and H. pylori do not normally cause such an acute symptom onset, and infectious organisms do not normally cause bleeding of the stomach lining. A combination of calcium carbonate and magnesium is commonly found in antacids. Overdose of antacids can result in irregular heartbeat, poor balance, shallow, rapid breathing and stupor (lack of alertness).

As you are walking in the park, a huge black Labrador (dog) runs up to you and places his paws on your shoulders. Immediately your heart starts racing, you feel palpations and anxiety, and your hands become a little shaky. The nurse knows that this response is primarily caused by A) fear of dogs that make you feel like your chest is being tightened and that you have lost control of the situation. B) increased levels of glucocorticoids by the adrenal glands that result in an increase in epinephrine level. C) response of the cholinergic muscarinic receptors on innervational targets of postganglionic fibers. D) stimulation of the release of β2-adrenergic receptors, which will open the airway and increase oxygenation.

B Any situation sufficiently stressful to evoke increased levels of glucocorticoids also increases epinephrine levels. Increased epinephrine levels results in tachycardia, palpitations, anxiety, and tremors. Cholinergic muscarinic receptors and β blockers do not help in times of immediate stress.

A nurse educator is orientating new nurses to a renal unit of the hospital. Which of the following teaching points should the nurse include as part of a review of normal glomerular function? A) "Nephrons are delicate structures that cannot endure the high pressure that exists in capillary beds elsewhere in the body." B) "Glomerular filtrate is very similar in composition to blood plasma found elsewhere in circulation." C) "Dilation of the afferent arteriole allows more blood into the nephron and increases the glomerular filtration rate." D) "The glomerulus is located between an arteriole and a venule that work together to regulate blood flow."

B Because filtration achieves the removal of nearly all blood proteins, the glomerular filtrate is nearly identical to plasma. Nephrons require high pressure to function, and dilation decreases the glomerular filtration rate. The glomerulus is located between two arterioles.

Which of the following clients' diagnostic blood work is most suggestive of chronic kidney disease (CKD)? A) A client with high pH; low levels of calcium; and low levels of phosphate B) A client with low vitamin D levels; low calcitriol levels; and elevated parathyroid hormone (PTH) levels C) A client with low bone density; low levels of calcium; and low levels of phosphate D) A client with low potassium levels; low calcitriol levels; and increased PTH levels

B CKD is associated with low vitamin D and calcitriol levels, which induces increased PTH production. CKD is also associated with acidosis (low pH), high levels of phosphate, and hyperkalemia.

The mother of a 19-week-old infant has brought her baby in for assessment to a pediatrician because of the baby's persistent weight loss and diarrhea. An intestinal biopsy has confirmed a diagnosis of celiac disease, and the child's mother is anxious to know what caused the disease. Which of the following aspects of the etiology of celiac disease would underlie the explanation that the physician provides? A) Bacterial or chemical invasion of the peritoneum leads to decreased nutrient absorption and transport. B) An inappropriate T-cell-mediated response results in increased levels of antibodies and an inflammatory response. C) Neurogenic or muscular inhibition of peristalsis results in inappropriate motility of ingested food in the lower small intestine and the colon. D) Inability to process or absorb the fat content of breast milk results in malnutrition and deficiency of fat-soluble vitamins.

B Celiac disease is rooted in an inappropriate immune response that initiates an inflammatory response, resulting in loss of absorptive villi. Bacterial or chemical invasion of the peritoneum is associated with peritonitis, while inhibition of peristalsis is associated with obstructions. An inability to process or absorb fat is associated with malabsorption syndrome.

During male ejaculation, which of the following statements addresses why sperm is not normally seen inside the bladder? A) The parasympathetic nervous system keeps the seminal fluid inside the urethra. B) The musculature of the trigone area, bladder neck, and prostatic urethra contract at the same time. C) With ejaculation, the male expels some urine along with the seminal fluid to wash any extra sperm out of the bladder. D) The detrusor muscle relaxes allowing for the closing of the sphincter at the base of the bladder.

B During male ejaculation, which is mediated by the SNS, the musculature of the trigone area and that of the bladder neck and prostatic urethra contracts and prevents the backflow of seminal fluid into the bladder.

While assessing a patient with urosepsis, the ICU nurse notes the patient's BP is 80/54; HR 132; RR 24; and pulse oximetry 89% on 6 lpm O2. Over the last hour, the patient's urine output is 15 mL. When explaining to a new graduate nurse, the nurse will emphasize that the patient's status may relate to that A) the infection is deep inside the kidney, and it will take a long time for the antibiotics to kill the bacteria. B) the patient's sympathetic nervous system has been stimulated that has resulted in vasoconstriction of the afferent arteriole, which causes a decrease in renal blood flow. C) the glomerular filtration system gets overwhelmed in times of stress (like infections) and can become clogged with waste material from the bacteria. D) the ability to transport substances from the tubular fluid into the peritubular capillaries becomes impaired, which results in fluid being forced out of capillaries into the glomerulus.

B During periods of strong sympathetic stimulation, such as shock, constriction of the afferent arteriole causes a marked decrease in renal blood flow and thus glomerular filtration pressure. Consequently, urine output can fall almost to zero.

A patient has recently been diagnosed with H. pylori gastritis. The nurse knows that this form of gastritis is usually treated with a combination of an antibiotic and A) antianxiety medications. B) proton pump inhibitors. C) lactulose, to reduce the blood ammonia levels. D) calcium carbonate, an antacid.

B H. pylori is associated with an increased risk of gastric adenocarcinoma, gastric atrophy, and peptic ulcer. It is less likely to contribute to IBD, esophagitis, or diverticular disease. Eradication of H. pylori is difficult. Treatment requires a combination therapy that includes the use of antibiotics and a proton pump inhibitor. The proton pump inhibitors have direct antimicrobial properties against H. pylori. Antianxiety medications will not kill the bacteria. H. pylori is not associated with elevated blood ammonia levels. Calcium carbonate is usually given to relieve heartburn caused by GERD.

Four weeks after returning from a tropical vacation, a 40-year-old man has presented to the emergency department with malaise, nausea, and "yellow eyes." Serology has confirmed a diagnosis of hepatitis A (HAV), to the shock of the client. What teaching is most appropriate for this client? A) "You can expect these symptoms to disappear after about 2 months, but you'll be a carrier of the disease indefinitely." B) "A vaccine before your trip would have prevented this, but be assured your body will rid itself of the virus in time." C) "You likely came in contact with blood or body fluids at some point, and you'll have to ensure no one is subsequently exposed to your own blood or body fluids." D) "You likely got this by way of what we call the 'fecal-oral' route; you will have chronic hepatitis now, but the symptoms can be controlled with medication."

B HAV is normally self-limiting and does not result in chronic hepatitis or carrier status. A vaccine is available, and the fecal-oral route of transmission, rather than contact with blood and body fluids, is typical.

A male patient comes to the clinic asking to speak to a health care provider privately. He reveals that he had shared a needle/syringe with a prostitute (shooting up cocaine) and then had unprotected sex. Upon questioning, it was revealed that the patient had not had any immunization for hepatitis B. Which of the following medications would the nurse anticipate administering today to this patient? A) Tenofovir disoproxil fumarate plus emtricitabine B) Hepatitis B immunoglobulin (HBIG) C) Hepatitis C immunoglobulin (HCIG) D) Hepatitis A vaccine

B HBIG is used as an adjunct to hepatitis B vaccine for postexposure immunoprophylaxis to prevent HBV infections in high-risk populations. Tenofovir disoproxil fumarate plus emtricitabine is for HIV preexposure coverage. There is no hepatitis C vaccine available on the market yet. Hepatitis A vaccine is for HAV, which is usually spread from fecal-oral routes.

A middle-aged male walks into the emergency department complaining of chest pain radiating to the neck, shortness of breath, and nausea. His heart rate is 120 and BP is 94/60. The ED physician recognizes the patient is having an acute MI with decreased cardiac output. The nurse identifies the nausea to be in response to A) the patient not having a very high pain tolerance. B) hypoxia exerting a direct effect on the chemoreceptor trigger zone. C) the patient not having digested his meal completely. D) fear of having to make major lifestyle changes.

B Hypoxia exerts a direct effect on the vomiting center (chemoreceptor trigger zone), producing nausea and vomiting. This direct effect probably accounts for the vomiting that occurs during periods of decreased cardiac output, shock, and environmental hypoxia. We are given no information about the patient's pain tolerance, when he last had a meal, or his routine lifestyle. This patient is going into shock (rapid pulse, low BP) that can result in shunting of blood away from the gut and other organs. During shock, the priority organs for oxygenation include the heart, brain, lungs, and kidneys.

Which of the following clients is most clearly displaying the signs and symptoms of irritable bowel disease (IBD)? A) A 32-year-old mother who complains of intermittent abdominal pain that is worse during her menstrual period B) A 51-year-old male who states that his stomach pain is in his lower abdomen, "comes and goes," and "feels more like a cramp than a dull ache" C) A 44-year-old man who works the evening shift at a factory and who states that his lower abdominal pain is much worse at night than during the day D) A 24-year-old man who has a stressful job but whose diarrhea and cramping do not worsen during periods of high stress

B IBD is commonly manifested as intermittent lower abdominal pain that feels like cramping. Defecation normally relieves the pain, and symptoms are normally not present at night or during sleep. Stress commonly exacerbates symptoms.

Which of the following statements best conveys an aspect of the role of cerebrospinal fluid (CSF)? A) It provides physical protection for the brain and ensures that leukocytes and erythrocytes are evenly distributed in the CNS. B) CSF cushions the brain and provides a near-water medium for diffusion of nutrients. C) CSF distributes plasma proteins throughout the superficial gray matter of the CNS. D) It ensures that the high metabolic and oxygenation needs of the brain are met, as well as absorbing physical shocks.

B In addition to providing a cushion for the CNS, CSF provides a medium that is 99% water in which nutrients, electrolytes, and wastes can be diffused. It is not centrally involved in the distribution of oxygen, plasma proteins, or blood cells.

A nurse is collecting a urine specimen prior to measuring the albumin level in a client's urine. A colleague questions the rationale for the test, stating, "I thought albumin was related to liver function, not kidney function." How can the nurse best respond to this statement? A) "Urine should normally be free of any proteins, and albumin is one of the more common proteins to be excreted in chronic renal failure." B) "Urine albumin levels are useful for diagnosing diabetic kidney disease." C) "A urine dipstick test will tell us exactly how much albumin is being spilled by the client's kidneys." D) "A urine test for albumin allows us to estimate the client's GFR quite accurately."

B In clients with diabetes, albumin tests are useful adjunctive test of nephron injury and repair. Urine is not normally completely free of proteins, and a urine dipstick does not allow for the quantification of how much albumin is in a sample. Albumin tests do not allow for an accurate indirect indication of GFR.

A middle-aged woman is brought to the emergency room after a minor auto accident. Her gait is staggering and unsteady; her speech is slurred; and she displays slight nystagmus. The police officer who brought her in says she has not been drinking. Her blood pressure is very high. Which of the following health problems most likely underlies her present state? A) Multiple sclerosis B) Guillain-Barré syndrome C) Myasthenia crisis D) Cerebellar damage caused by a cerebrovascular accident

D Cerebellar damage can resemble the effects of alcohol, because alcohol affects the cerebellum. This woman has not been drinking, but her high blood pressure puts her at risk for cerebrovascular accident.

Following an automobile accident where the patient had a traumatic amputation of his lower leg and lost greater than 40% of his blood volume, he is currently not producing any urine output. The nurse bases this phenomena on which of the following humoral substances responsible for causing severe vasoconstriction of the renal vessels? A) Aquaproin-2 channels B) Angiotensin II and ADH C) Renin and potassium ions D) Albumin and norepinephrine

B Increased sympathetic activity causes constriction of the afferent and efferent arterioles and thus a marked decrease in renal blood flow. Intense sympathetic stimulation can produce marked decreases in renal blood flow and GFR. Humoral substances, including angiotensin II, ADH, and endothelins, produce vasoconstriction of renal blood flow. Aquaporin-2 channels, potassium ions, and albumin do not have vasoconstriction properties.

A 22-year-old student has developed a fever and diarrhea while on a backpacking trip in Southeast Asia. His oral temperature is 101.4°F. The diarrhea is bloody, frequent, and small in volume. These clinical manifestations are sufficiently distressing that he is visiting a local medical clinic in the area. Which of the following diagnoses best characterizes this health problem? A) Noninflammatory diarrhea B) Inflammatory diarrhea C) Factitious diarrhea D) Secretory diarrhea

B Inflammatory diarrhea is often characterized by small-volume diarrhea that is bloody and accompanied by a fever. Noninflammatory diarrhea is normally larger in volume and not bloody. Factitious diarrhea is normally attributable to laxative use, and secretory diarrhea is associated with increased secretory processes of the bowel; neither is likely to produce bloody stool.

A 40-year-old man who uses heroin intravenously was diagnosed with hepatitis C (HCV) 1 year ago and is now considered to have chronic viral hepatitis. Which of the following statements by the client to his care provider would warrant correction? A) "I know the medications to treat this aren't fantastic, but at least there are some options for controlling the virus." B) "It's at least a bit reassuring that my liver isn't undergoing damage when I'm not experiencing symptoms." C) "Even though I'm sick, at least I won't feel sick most of the time." D) "I'm not looking forward to all the side effects of the drug treatments for my HCV, but I hope I don't end up needing a liver transplant."

B Liver damage persists both during symptomatic and asymptomatic periods of acute viral hepatitis. Medications do exist for treatment of the disease, and asymptomatic periods are more common than symptomatic episodes. Side effects of drug treatment are common, and transplant is a potential end-stage treatment option.

A 51-year-old woman diagnosed with a cerebrovascular accident (CVA) 5 months prior is distressed that she has had several recent episodes of urinary incontinence. She has asked her nurse practitioner why this is the case. Which of the following statements best captures the fact that would underlie the nurse's response to the client? A) Neurological diseases like MS often result in flaccid bladder dysfunction. B) She may be unable to sense her bladder filling as a result of her MS. C) Lesions to the basal ganglia or extrapyramidal tract associated with MS inhibit detrusor contraction. D) Pathological reductions in bladder volume brought on my MS necessitate frequent micturition.

B MS may result in neurogenic bladder characterized by an inability to sense filling and consequent incontinence. She is not demonstrating the signs of a flaccid bladder, and lesions to the basal ganglia or extrapyramidal tract are associated with Parkinson disease, not MS. Her disease is unlikely to directly reduce bladder volume.

A nurse in an acute medical unit of a hospital has admitted a 62-year-old female from the emergency department who has been diagnosed with acute pyelonephritis. Which of the following statements most accurately conveys an aspect of the knowledge base that the nurse needs to perform adequate care and teaching? A) Most cases of acute pyelonephritis are attributable to poorly controlled hypertension. B) Flank pain, dysuria, and nausea and vomiting are likely assessment findings. C) The infection in the kidney is most likely a manifestation of a systemic infection. D) Imaging tests are likely to reveal scarring and deformation of the renal calices and pelvis.

B Manifestations of acute pyelonephritis include pain, frequency, urgency, dysuria, nausea, and vomiting. Chronic rather than acute pyelonephritis is often caused by hypertension, while most cases are caused by ascending bacteria, not systemic infections. Scarring is more commonly a result of chronic pyelonephritis.

A 15-year-old who has just been diagnosed with type 1 diabetes says she read on the Internet that diabetes is the leading cause of acquired blindness among Americans. She asks you if she will lose her sight. In addition to explaining that new treatment technologies are being worked on every day, which of the following would be the most appropriate response? A) Reassure her that only type 2 diabetes is a risk factor for blindness. B) Explain that almost all people with type 1 diabetes do experience some degree of vision loss. C) Tell her to expect that she will begin to lose her eyesight by the time she is 25. D) Tell her there is about a 50%chance that she will suffer some diabetes-related sight loss by the time she is 50.

B Nearly all people with type 1 diabetes and more than 60% of people with type 2 diabetes have some degree of retinopathy. Pregnancy, puberty, and cataract surgery can accelerate these changes.

To reduce hepatic blood flow and decrease portal pressures in persons with cirrhosis, the nurse should be prepared to administer which of the following medications? A) Bevacizumab, an angiogenesis inhibitor B) Octreotide, a long-acting synthetic analog of somatostatin C) Filgrastim, granulocyte colony-stimulating factor (G-CSF) analog D) Diltiazem (Cardizem), a calcium channel blocker

B Ocetrotide, a long-acting synthetic analog of somatostatin, reduces splanchnic and hepatic blood flow and portal pressures in persons with cirrhosis. Bevacizumab was the first angiogenesis inhibitor that was shown to slow tumor growth and, more importantly, to extend the lives of patients with some cancers. Filgrastim rapidly reverses neutropenia and maintains normal ANC in patients with HIV infection. Diltiazem is used to treat a variety of conditions, such as high blood pressure, migraines, and Raynaud disease.

Which of the following medications would the nurse anticipate being prescribed for the renal failure patient who has hyperphosphatemia? A) Vitamin D (calcitriol) B) Calcium carbonate C) Levothyroxine (Synthroid) D) Sensipar (Cinacalcet)

B Phosphate-binding antacids (aluminum salts, calcium carbonate, or calcium acetate) may be prescribed to decrease the absorption of phosphate from the GI tract.

A 60-year-old man has been diagnosed with renal calculi after repeated episodes of excruciating flank pain in recent weeks. The man states that, "I don't know how this could happen to me, since I'm so careful about eating a healthy diet." What is the most appropriate response to the man's statement? A) "Your diet may have played a part in this, but in fact, genetics are likely primarily to blame." B) "What you eat can influence your risk of stone formation, but many other factors like hormones and your metabolism are involved." C) "You likely don't need to change your diet, but now that you have stones in one kidney, you're at very high risk of growing them in the other kidney." D) "Your diet might be normally healthy, but high intake of normally beneficial minerals like calcium and magnesium can lead to stones."

B Renal calculi have a complex etiology that includes diet, but also many other metabolic and endocrine factors, among others. Genetics are not identified as a contributor, and mineral intake is not likely to be the sole factor. Stone formation is normally unilateral.

A 68-year-old African American man who has smoked for at least 50 years reports that lately he feels as though food is getting stuck in his throat. At first, this was a problem just with dry food, but now his morning oatmeal is getting "stuck." On questioning, he reports drinking at least three alcoholic beverages nearly every day. His problem is most likely A) achalasia. B) squamous cell carcinoma of the esophagus. C) dysphagia secondary to scleroderma. D) gastrointestinal reflux disease.

B Squamous cell carcinoma of the esophagus is the seventh leading cause of cancer death among men, particularly black men; mean age at diagnosis is 67 years. Alcohol and tobacco use are the main risk factors for this cancer, and dysphagia is a common presenting complaint. An esophageal motility disorder involves the smooth muscle layer of the esophagus and the lower esophageal splincter (LES). Achalasia is characterized by difficulty swallowing and regurgitation. GERD (gastroesophageal reflux disease) is a condition that causes the esophagus to become irritated and inflamed. Clients with GERD usually feel a burning in the chest or throat called heartburn. Sometimes, they taste stomach fluid in the back of the mouth.

When reviewing the purpose/action of neurotransmitters as they interact with different receptors, the nursing instructor gives an example using acetylcholine. When acetylcholine is released at the sinoatrial node in the right atrium of the heart, it is A) positively charged. B) inhibitory. C) overstimulated. D) dormant.

B The action of a transmitter is determined by the type of receptor to which it binds. Acetylcholine is excitatory when it is released at a myoneural junction, and it is inhibitory when it is released at the sinoatrial node in the heart.

The nurse knows which of the following phenomena listed below is an accurate statement about axonal transport? A) Anterograde and retrograde axonal transport allow for the communication of nerve impulses between the neuron and the central nervous system (CNS). B) Materials can be transported to the nerve terminal by either a fast or slow component. C) The unidirectional nature of the axonal transport system protects the CNS against potential pathogens. D) Axonal transport facilitates the movement of electrical impulses but precludes the transport of molecular materials.

B The bidirectional axonal transport system allows for the transport of molecular materials (as opposed to electrical impulses); anterograde transport has both slow and fast components.

A 21-year-old male is brought to the ED following a night of partying in his fraternity. His friends found him "asleep" and could not get him to respond. They cannot recall how many alcoholic beverages he drank the night before. While educating a student nurse and the roommates in the fraternity, the nurse begins by explaining that alcohol is A) water-soluble compound that is easily absorbed by the gastric lining of the stomach. B) very lipid soluble and rapidly crosses the blood-brain barrier. C) able to reverse the transport of some substances to remove them from the brain. D) very likely to cause sedation, and therefore the patient just needs to sleep it off.

B The blood-brain barrier prevents many drugs from entering the brain. Most highly water-soluble compounds are excluded from the brain. Many lipid-soluble molecules cross the lipid layers of the blood-brain barrier with ease. Alcohol, nicotine, and heroin are very lipid soluble and rapidly enter the brain. Alcohol toxicity can kill patients, especially if they are not used to consuming beverages. These patients should never be left alone to "sleep it off."

While assessing a critically ill patient in the emergency department, the nurse notes on the cardiac monitor an R-on-T premature ventricular beat that develops into ventricular tachycardia (VT). Immediately, the patient became unresponsive. The nurse knows that based on pathophysiologic principles, the most likely cause of the unresponsiveness is A) metabolic acidosis that occurs spontaneously following any dysrhythmias. B) interruption of the blood/oxygen supply to the brain. C) massive cerebrovascular accident (CVA) resulting from increased perfusion. D) a blood clot coming from the heart and occluding the carotid arteries.

B The brain receives 15% to 20% of the total resting cardiac output and consumes 20% of its oxygen. The brain cannot store oxygen or engage in anaerobic metabolism. An interruption of blood or oxygen supply to the brain rapidly leads to clinically observable signs and symptoms. Unconsciousness occurs almost simultaneously with cardiac arrest. Metabolic acidosis will occur later in the cardiac arrest but not immediately and is not responsible for the patient's unresponsiveness. CVAs can be caused by thrombosis formation or plaque occlusions, but it is not the primary reason for unconsciousness in VT.

A nurse on a neurology unit is assessing a female brain-injured client. The client is unresponsive to speech, and her pupils are dilated and do not react to light. She is breathing regularly, but her respiratory rate is 45 breaths/minute. In response to a noxious stimulus, her arms and legs extend rigidly. What is her level of impairment? A) Delirium B) Coma C) Brain death D) Vegetative state

B The continuum of loss of consciousness is marked by the degree of client's responsiveness to stimuli, in addition to the preservation of brain stem reflexes. Since this client still exhibits a pain response (the extended arms and legs indicate decerebrate posturing), even though her pupils are not responsive to light, she has sustained sufficient brain function that she fails to qualify as being brain dead or in a vegetative state.

A 22-year-old female college student is shocked to receive a diagnosis of myasthenia gravis. What are the etiology and most likely treatment for her health problem? A) Autoimmune destruction of skeletal muscle cells; treatment with intensive physical therapy and anabolic steroids B) A decline in functioning acetylcholine receptors; treatment with corticosteroids and intravenous immunoglobulins C) Cerebellar lesions; surgical and immunosuppressive treatment D) Excess acetylcholinesterase production; treatment with thymectomy

B The etiology of myasthenia gravis involves a deficiency of acetylcholine receptors at neuromuscular junctions. Treatment can include corticosteroid therapy and intravenous immunoglobulins. Destruction of skeletal muscle cells, cerebellar lesions, and excess acetylcholinesterase are not noted to underlie the disease.

Which of the following data would a clinician consider as most indicative of acute renal failure? A) Alterations in blood pH; peripheral edema B) Increased nitrogenous waste levels; decreased glomerular filtration rate (GFR) C) Decreased serum creatinine and blood urea nitrogen (BUN); decreased potassium and calcium levels D) Decreased urine output; hematuria; increased GFR

B The hallmark of acute renal injury is azotemia, an accumulation of nitrogenous wastes such as creatinine, urea nitrogen, and uric acid, plus a decrease in the GFR of the kidneys. While pH alterations, edema, electrolyte imbalances, and decreased urine output may accompany acute renal failure, they are all potentially attributable to other pathologies. Creatinine, GFR, and BUN would unlikely rise during renal failure.

A 55-year-old man has made an appointment to see his family physician because he has been awakening three to four times nightly to void and often has a sudden need to void with little warning during the day. What is the man's most likely diagnosis and possible underlying pathophysiological problem? A) Stress incontinence due to damage to CNS inhibitory pathways B) Overactive bladder that may result from both neurogenic and myogenic sources C) Overactive bladder due to intravesical pressure exceeding urethral pressure D) Overflow incontinence that can result from displacement of the angle between the bladder and the posterior proximal urethra

B The man's complaints are typical of overactive bladder, a condition that can result from the interaction of both the nervous control of bladder emptying and the muscles of the bladder itself. His symptoms are not characteristic of stress incontinence, and when intravesical pressure exceeds, urethral pressure overflow incontinence results. The angle between the bladder and the posterior proximal urethra is more commonly a factor in the continence of females.

A 9-year-old boy has been brought to the emergency department by his father who is concerned by his son's recent fever, stiff neck, pain, and nausea. Examination reveals a petechial rash. Which of the following assessment questions by the emergency room physician is most appropriate? A) "Is your son currently taking any medications?" B) "Has your son had any sinus or ear infections in the last little while?" C) "Does your son have a history of cancer?" D) "Was your son born with any problems that affect his bone marrow or blood?"

B The most common symptoms of acute bacterial meningitis are fever and chills; headache; stiff neck; back, abdominal, and extremity pains; and nausea and vomiting. Risk factors associated with contracting meningitis include otitis media and sinusitis or mastoiditis. Particular medications, a history of neoplasm, and hematopoietic problems would be unlikely to relate directly to his symptoms of meningitis.

A 53-year-old woman with a history of chronic alcohol abuse but without visible jaundice comes to the clinic complaining of nausea and weakness. She admits to taking acetaminophen for persistent headaches but denies exceeding the recommended daily dose; she has not taken any other medications. She is suspected of having acetaminophen toxicity. Which of the following diagnostic test findings would implicate a different cause of her symptoms? A) Normal serum acetaminophen level B) Elevated serum HBsAg level C) Evidence of steatosis on liver biopsy tissue sample D) Hypoglycemia

B The presence of HBsAg would suggest that this woman is in the prodromal phase of hepatitis B infection. A normal serum acetaminophen level does not preclude toxicity if the drug is taken over a period of time. Steatosis is fatty infiltration of the liver. Steatosis is often but not exclusively an early histological feature of alcoholic liver disease (therefore, with chronic alcohol abuse, the nurse should expect the patient may have steatosis). Drinking heavily without eating can block your liver from releasing stored glucose into your bloodstream, causing hypoglycemia.

During a family picnic, a relative of a nurse asks what he should do if there is blood in his urine and some pain in his lower abdomen. The best advice the nurse could give this family member would be for him to A) go to the emergency room right away. B) get an appointment with his family doctor. C) wait and see if it goes away without treatment. D) increase his intake of cranberry juice and other fluids.

B There are many causes of blood in the urine (infection, kidney stones, cancer, etc.). Kidney cancer can be a silent disorder with symptoms occurring late into the disease process. Any blood in the urine should be referred to the physician for further testing (ultrasound, CT scan, MRI, etc.).

A 40-year-old man who is morbidly obese and leads a sedentary lifestyle has recently been diagnosed with type 2 diabetes. Which of the following aspects of the man's obesity likely contributed to his new health problem? A) The low metabolic needs of adipose tissue mimic a hypoglycemic state and suppress insulin secretion. B) Free fatty acids contribute to problems such as beta cell dysfunction and insulin resistance. C) Fat tissue initiates glycogenolysis and reliance on glycogen release rather than metabolism of free glucose. D) Triglyceride deposits in the pancreas result in damage to beta cells.

B Type 2 diabetes in obese people is thought to link to the actions of free fatty acids, which include beta cell dysfunction (lipotoxicity), insulin resistance, glucose underutilization, and the accumulation of FFAs and triglycerides reduce hepatic insulin sensitivity

A clinician is conducting an assessment of a male client suspected of having a disorder of motor function. Which of the following assessment findings would suggest a possible upper motor neuron (UMN) lesion? A) The client has decreased deep tendon reflexes. B) The client displays increased muscle tone. C) The client's muscles appear atrophied. D) The client displays weakness in the distal portions of his limbs.

B UMNs typically produce increased muscle tone, while hyporeflexia, muscle atrophy, and weakness in the distal portion of limbs are more commonly indicative of LMN lesions.

An 82-year-old resident of a long-term care facility with a recent history of repeated urinary tract infections and restlessness is suspected of having urinary retention. Which of the following actions by the care team is most appropriate? A) Uroflowmetry to determine the rate of the client's urine flow B) Ultrasound bladder scanning to determine the residual volume of urine after voiding C) Renal ultrasound aimed at identifying acute or chronic kidney disease D) Urinalysis focusing on the presence of or absence of microorganisms, blood, or white cells in the man's urine

B Ultrasound bladder scanning yields a fast and noninvasive indication of whether or not an individual is adequately emptying his or her bladder with each void. Uroflowmetry would be less indicative of whether the man is retaining, and renal ultrasound would address deficits in urine production rather than bladder emptying. Urinalysis would be useful in the diagnosis of infections and/or renal issues more than deficiencies in bladder emptying.

A frantic mother brings her young child into the emergency department. She states that during the evening bath, she noticed a large mass in her child's abdomen. After diagnostic testing, the pediatrician tells the parents that their child has Wilms tumor, stage IV. After the doctor leaves the room, the parents ask the nurse, "What does this mean?" The nurse will respond, "Your child ('s) A) "has cancer in his stomach." B) "has cancer in the kidney that has spread most likely to his lungs." C) "will need to undergo surgery to remove both kidneys and then go on dialysis." D) "tumor can be easily treated with chemotherapy. We will start this soon."

B Wilms tumor usually is a solitary mass that occurs in any part of the kidney. It usually is sharply demarcated and variably encapsulated. Stage IV means the tumors have undergone hematogenous metastasis, most commonly involving the lung. Treatment involves surgery, chemotherapy, and sometimes radiation therapy. Long-term survival rates are good (90%) for stages I to III.

Following an oral glucose tolerance, a 36-year-old mother of four has been diagnosed with gestational diabetes mellitus (GDM), a problem that was not present in any of her previous pregnancies. What should her primary care provider tell her about this new health problem? A) "This diabetes is unlikely to persist after you give birth, but the main risk is that your baby will likely be born with diabetes." B) "Your baby could become too large or have low blood sugars if we're not vigilant about controlling your sugars." C) "We'll monitor this closely and begin insulin therapy as soon as possible." D) "This is likely a result of your liver releasing too much fat, rather than your pancreas not secreting insulin."

B Women with GDM are at higher risk for complications of pregnancy, mortality, and fetal abnormalities. Fetal abnormalities include macrosomia, hypoglycemia, hypocalcemia, polycythemia, and hyperbilirubinemia. GDM often persists as type 2 diabetes after delivery, and the baby does not face a significantly higher risk of developing diabetes. Nutrition therapy would precede insulin therapy, and GDM involves a pancreatic etiology.

A 9-year-old boy has been diagnosed with the nephrotic syndrome. Place the following stages in the development of his health problem in ascending order. Use all the options. A) Hypoalbuminemia B) Increased glomerular membrane permeability C) Decreased colloidal osmotic pressure D) Proteins escape from the plasma to the glomerular filtrate E) Accumulation of fluid in the interstitial tissue (edema)

B D A C E The pathophysiology of the nephrotic syndrome involves damaged glomeruli becoming increasingly permeable to protein, allowing more protein into the glomerular filtrate. Massive proteinuria results, leading to hypoalbuminemia. Generalized edema, which is the hallmark of nephrotic syndrome, results from the loss of colloidal osmotic pressure of the blood with subsequent accumulation of fluid in the interstitial tissues.

Which of the following statements about mesangial cells within the glomerulus is accurate? Select all that apply. A) cover the entire amount of endothelial cells contained within the capillaries. B) have phagocytic properties that remove macromolecular materials. C) exhibit vasodilator properties to assist with increase in blood flow in times of stress. D) enlarge (hyperplasia) in response to glomerular diseases. E) are coiled and drain Bowman capsule.

B, D Mesangial cells possess phagocytic properties and remove macromolecular materials that enter the intercapillary spaces. Mesangial cells also exhibit contractile properties in response to neurohumoral substances and are thought to contribute to the regulation of blood flow through the glomerulus. Mesangial hyperplasia and increased mesangial matrix occur in a number of glomerular diseases. The nephron tubule, called the proximal convoluted tubule, is coiled and drains the Bowman capsule

Which of the following patients scheduled for an interventional radiology procedure requiring administration of radiocontrast dye would be considered at high risk for nephrotoxicity? Select all that apply. A) A 14-year-old with severe abdominal pain B) A 25-year-old with a history of glomerular nephritis who is complaining of severe flank pain C) A 67-year-old diabetic undergoing diagnostic testing for new-onset proteinuria D) A 45-year-old with elevated liver enzymes possibly due to fatty liver cirrhosis E) A 53-year-old male undergoing biopsy for a suspicious "spot" on his chest x-ray

B, C Radiocontrast media-induced nephrotoxicity is thought to result from direct tubular toxicity and renal ischemia. The risk for renal damage caused by radiocontrast media is greatest in older adults and those with preexisting kidney disease, volume depletion, diabetes mellitus, and recent exposure to other nephrotoxic agents.

Following an injury where a child hit his head from a fall, the CT scan reveals a contusion that the doctor classifies as a moderate brain injury. Which of the following manifestations will the nurse more than likely assess on this child that support this diagnosis? Select all that apply. A) Coma with total paralysis B) Periods of unconsciousness C) Aphasia at times D) Nuchal rigidity E) Weakness or slight paralysis affecting one side of the body

B, C, E Moderate brain injury is characterized by a period of unconsciousness and may be associated with focal manifestations such as hemiparesis (weakness or slight paralysis affecting one side of the body), aphasia, and cranial nerve palsy. Coma with total paralysis is seen in severe brain injury. Nuchal rigidity is a classic sign of meningitis.

Which of the following individuals are likely to display identified risk factors for the development of lower urinary tract obstruction? Select all that apply. A) A 32-year-old woman who had a healthy delivery of her third child 4 months ago B) A 68-year-old man who has been diagnosed with benign prostatic hyperplasia (BPH) C) A 55-year-old man with diabetes who is receiving diuretic medications for the treatment of hypertension D) A 30-year-old woman who has been diagnosed with gonorrhea E) A 74-year-old woman who has developed a lower bowel obstruction following several weeks of chronic constipation F) A 20-year-old man who has spina bifida and consequent impaired mobility.

B, D, E, F BPH frequently obstructs the urethra, while sexually transmitted diseases, bowel obstructions, and spina bifida are also associated with physical blockages of the lower urinary tract. Postpartum women and individuals receiving diuretics would be more likely to be at risk for incontinence rather than urinary retention.

A nurse has noted the high incidence of urinary tract obstructions of a variety of etiologies. Which of the following individuals are at risk of developing urinary obstructions? Select all that apply. A) 43-year-old male with an acid-base imbalance secondary to malnutrition B) A 29-year-old female, pregnant for the first time C) A 69-year-old female with anemia secondary to insufficient erythropoietin production D) A 70-year-old male with benign prostatic hyperplasia (BPH) E) A 58-year-old male with renal calculi F) A 28-year-old male with a neurogenic bladder secondary to spinal cord injury

B, D, E, F Pregnancy, BPH, renal calculi, and neurogenic bladder are all identified contributors to urinary obstructions. Acid-base imbalances and impaired erythropoietin production are health problems with renal involvement but are less likely to contribute to urinary obstruction.

Which of the following lab results would be associated with abnormalities in kidney function? Select all that apply. A) An absence of protein in a urine sample B) Increased creatinine levels C) Urine gravity of 1.038 and normal serum creatinine levels D) Decreased blood urea nitrogen (BUN) level E) Detectable levels of glucose in a urine sample F) Elevated cystatin-C level.

B, E, F Increased creatinine and BUN are associated with abnormalities in renal function, as is the presence of glucose in a urine sample. Urine samples normally lack protein and have a specific gravity of 1.038 to 1.040. An elevated BUN coupled with normal creatinine is likely not attributable to impaired kidney function.

Following a 14-day course of broad-spectrum antibiotics for the treatment of sepsis, a 60-year-old woman has developed watery diarrhea. Her care team attributes this to likely elimination of normal intestinal flora by the antibiotics. What other phenomena is most likely accompanying her low levels of normal flora? A) Decreased mineral and nutrient absorption; decreased carbohydrate metabolism B) Decreased pH of the stomach; increased pH of the lower GI tract C) Decreased fermentation of undigestible dietary residue; decreased vitamin absorption D) Proliferation of vitamin K; lower GI bleeding

C Central among the functions of normal intestinal flora are the fermentation of dietary components that are not digestible and the facilitation of vitamin absorption. Mineral and nutrient absorption as well as carbohydrate metabolism are less likely to be affected, while vitamin K production would likely decrease somewhat. pH is unlikely to be affected.

A baseball player was hit in the head with a bat during practice. In the emergency department, the physician tells the family that he has a "coup" injury. How will the nurse explain this to the family so they can understand? A) "It's like squeezing an orange so tight that the juice runs out of the top." B) "Your son has a huge laceration inside his brain where the bat hit his skull." C) "Your son has a contusion of the brain at the site where the bat hit his head." D) "When the bat hit his head, his neck jerked backward causing injury to the spine."

C A direct contusion of the brain at the site of external force is referred to as a coup injury. Contrecoup injury (answer choice D) is the rebound injury on the opposite side of the brain. Answer choice B relates to lacerations that are usually not caused by a direct blow to the head. However, depending on how hard the head was hit with a bat, a hematoma could form as the brain strikes the rough surface of the cranial vault.

A nurse working in a busy orthopedic clinic is asked to perform the Tinel sign on a patient having problems in his hand/wrist. In order to test Tinel sign, the nurse should give the patient which of the following directions? A) "Stand tall, arms at your side, shut your eyes; place the tip of your index finger to your nose." B) "Hold your wrist in complete flexion; keep it in this position for 60 seconds; how does your hand feel after placing it in a neutral position?" C) "I'm going to tap (percuss) over the median nerve in your wrist; tell me what sensation you feel while I am doing this. Does the sensation stay in the wrist or go anywhere else?" D) "I'm going to tap this tuning fork and place it on the side of your thumb; then tell me what you are feeling in your hand and wrist."

C A positive Tinel sign will help diagnose carpal tunnel syndrome. The patient will have a tingling sensation radiating into the palm of the hand when lightly percussed over the median nerve at the wrist. Answer choice B relates to Phalen sign, which is also a test to help diagnose carpel tunnel syndrome. Answer choice D is not the medical test for carpal tunnel syndrome.

he clinical nurse educator on a nephrology unit of a large, urban hospital is orientating recent nursing graduates to the unit. Which of the following teaching points about acute tubular necrosis (ATN) should the educator include in the orientation session? A) "The cardinal signs of ATN are oliguria and retention of potassium, creatinine, and sulfates." B) "Ureteral and bladder outlet obstructions are often contributors to ATN." C) "Trauma, burns, and major surgery are common precursors to ATN." D) "Tubular epithelial cells are sensitive to ischemia and toxins, and damage is irreversible."

C ATN is often preceded by major surgery, burns, or trauma. Many cases of ATN are nonoliguric, and obstructions that are postrenal in nature are not common causes of ATN. Damage to tubular epithelial cells is not necessarily irreversible.

The geriatrician providing care for a 74-year-old man with diagnosis of Parkinson disease has recently changed the client's medication regimen. What is the most likely focus of the pharmacologic treatment of the man's health problem? A) Maximizing acetylcholine release from synaptic vesicles at neuromuscular junctions B) Preventing demyelination of the efferent cerebellar pathways C) Increasing the functional ability of the underactive dopaminergic system D) Preventing axonal degradation of motor neurons

C Antiparkinson drugs act by increasing the functional ability of the underactive dopaminergic system. The cerebellar pathways, acetylcholine levels, and axonal degradation are not components of the etiology of Parkinson disease.

Damage to which of the following areas of a nephron would most likely result in impaired secretion and reabsorption? A) Distal tubule B) Loop of Henle C) Proximal tubule D) Collecting tubule

C Approximately two thirds of the absorption and secretion that occur in the tubular system take place in the proximal tubule.

A male client complaining of chronic cramping, bloating, and diarrhea has been determined to have a deficiency in brush border enzymes within his small intestine. Which of the following meals or snacks high in carbohydrates and protein will likely exacerbate the client's signs and symptoms? A) Grapefruit and prunes B) Tossed salad with an oil and vinegar dressing C) Roast beef and a baked potato D) Tortilla chips and guacamole

C Brush border enzymes are primarily responsible for the metabolism of carbohydrates and proteins, substances best exemplified by a baked potato and roast beef, respectively. Of the distracters, choice C has both high protein and high carbohydrate content. Distracters A, B, and D are not high in both carbohydrates and proteins.

A toddler is displaying signs and symptoms of weakness and muscle atrophy. The pediatric neurologist suspects it may be a lower motor neuron disease called spinal muscular atrophy (SMA). The patient's family asks how he got this. The nurse will respond A) "This could result from playing in soil and then ingesting bacteria that are now attacking his motor neurons." B) "No one really knows how this disease is formed. We just know that in time, he may grow out of it." C) "This is a degenerative disorder that tends to be inherited as an autosomal recessive trait." D) "This is a segmental demyelination disorder that affects all nerve roots and eventually all muscle groups as well."

C SMA is a distinctive group of degenerative disorders involving LMNs that begins in childhood. Answer choice A relates to botulism. It is known which gene is involved in SMA. Answer choice D does not describe SMA.

A geriatrician and social worker are facilitating a family meeting for the children and wife of a 79-year-old man who has been diagnosed with Alzheimer disease. What goal of treatment will the clinicians most likely prioritize in their interactions with the family? A) Modest reversal of brain plaque formation and improved symptomatology through cholinesterase inhibitors B) Surgical treatment of the underlying ischemic changes that underlie the manifestations of the man's disease C) The use of medications such as donepezil and rivastigmine to slow the progression of the disease D) Cognitive and behavioral therapy to counteract the agitation, depression, and suspiciousness associated with Alzheimer disease

C Cholinesterase inhibitors like donepezil and rivastigmine may slow the progression of Alzheimer's but do not reverse the organic brain changes that characterize the etiology. Ischemic changes are associated with vascular dementia, and cognitive therapy is of limited use in treatment.

Which of the following individuals is likely to have the best prognosis for recovery from his or her insult to the peripheral nervous system? An adult A) who developed rhabdomyolysis and ischemic injury after a tourniquet application. B) who suffered a bone-depth laceration to the shoulder during a knife attack. C) who had his forearm partially crushed by gears during an industrial accident. D) who had nerves transected during surgery to remove a tumor from the mandible.

C Crushing-type injuries carry a higher possibility of nerve function recovery than do lacerations and insults resulting from ischemia.

A family physician is providing care for a 61-year-old obese male who has a history of diabetes and hypertension. Blood work has indicated that the man has a GFR of 51 mL/minute with elevated serum creatinine levels. Which of the following statements will the physician most likely provide the client in light of these results? A) "We will regularly monitor your kidney function, but most likely your kidneys will be able to compensate on their own and intervention is not required." B) "You likely have chronic kidney disease, and there may be urine in your blood until it is controlled." C) "Your chronic kidney disease has likely been caused by your diabetes and high blood pressure." D) "You're in kidney failure, and I'll be starting dialysis treatment immediately."

C Diabetes and hypertension are conditions that can cause chronic kidney disease (CKD). While the kidneys do have a remarkable ability to compensate for impaired function, this fact does not mean that treatment would not be undertaken. Hematuria is not a common manifestation of CKD, and the client's GFR of 51 mL/minute does not indicate kidney failure or the need for dialysis.

A teenager, exposed to West Nile virus a few weeks ago while camping with friends, is admitted with headache, fever, and nuchal rigidity. The teenager is also displaying some lethargy and disorientation. The nurse knows which of the following medical diagnoses listed below may be associated with these clinical manifestations? A) Rocky Mountain spotted fever B) Lyme disease C) Encephalitis D) Spinal infection

C Encephalitis represents a generalized infection of the parenchyma of the brain or spinal cord. A virus, such as West Nile virus, usually causes encephalitis although it may be caused by bacteria, fungi, and other organisms. Encephalitis is characterized by fever, headache, and nuchal rigidity. However, more often, people also experience neurologic disturbances, such as lethargy, disorientation, seizures, focal paralysis, delirium, and coma. Rocky Mountain spotted fever (answer choice A) is a tick-borne disease caused by the bacterium Rickettsia rickettsii and usually begins with a sudden onset of fever and headache. A rash may occur 2 to 5 days after fever onset. Lyme disease (answer choice B) is also a tick-borne disease. Typical symptoms include fever, headache, fatigue, and a characteristic skin rash called erythema migrans. As the disease progresses, the patient develops bouts of severe joint pain and swelling of the joint. Neurological problems may occur for weeks, months, or even years after the infection and may include inflammation of the membranes surrounding the brain (meningitis). Spinal infections (answer choice D) can be thought of as a spectrum of diseases comprising spondylitis, discitis, spondylodiscitis, pyogenic facet arthropathy, and meningitis.

A nurse practitioner is providing care for a male client with a long-standing hiatal hernia. Which of the following statements most accurately captures an aspect of the pathophysiology of hiatal hernias? A) Paraesophageal hiatal hernias are common and are normally not treated if the client is asymptomatic. B) The root causes of hiatal hernias are normally treatable with medication. C) If esophageal acid clearance is impaired, esophagitis can result. D) An incompetent pyloric sphincter and high-fat diet are commonly implicated in the development of hiatal hernias.

C Erosive esophagitis can be a complication of hiatal hernias if esophageal acid clearance is significantly impaired. Paraesophageal hiatal hernias are more serious than the sliding variety and require treatment. The root cause of hiatal hernias, herniation of the stomach through the diaphragm, is not normally amenable to treatment with medication. The pyloric sphincter is not associated with hiatal hernias.

ollowing a history of gastric pain and an endoscopy, a client has been diagnosed with a duodenal peptic ulcer. Which of the following teaching points should his caregiver provide? A) "While your diet most certainly contributed to this problem, the good news is that changing your diet can help solve it." B) "Ulcers like yours do not penetrate all layers of the stomach or duodenum, so you don't have to worry about losing too much blood." C) "Your family history, your smoking history, and NSAID use may all have contributed to this problem." D) "While there aren't really any effective medications for these ulcers, changes in lifestyle can keep them well controlled."

C Family history, NSAID use, and smoking have all been identified as contributing factors in the development of peptic ulcers. Diet therapy has not been shown to be effective, and duodenal peptic ulcers are more common than the gastric variant. Perforation occurs when an ulcer erodes through all layers of the stomach or duodenum wall. When perforation occurs in older adults, their mortality is significantly increased. Effective medication regimens are available with antacids, H2-receptor antagonists or proton pump inhibitors being the most common medications used.

Following a spinal cord injury suffered in a motor vehicle accident, a 22-year-old male has lost fine motor function of his fingers and thumb but is still able to perform gross motor movements of his hand and arm. Which of the following components of his white matter has most likely been damaged? A) The inner layer (archilayer) B) The middle layer (paleolayer) C) The outer layer (neolayer) D) The reticular formation

C Fine manipulation skills are the domain of the outer layer, or neolayer, of the tract systems. The inner and middle layers and the reticular formation are not noted to be responsible for these functions.

Following several days of intermittent upper right quadrant pain, a 29-year-old obese, Native American woman has been diagnosed with cholelithiasis. The nurse at the clinic has taught the client about the pathophysiology and contributing factors to her health problem, as well as some of the likely treatment options. Which of the following statements by the client demonstrates a sound understanding of her diagnosis? A) "All in all, I guess this is a result of the fact that I've been eating a diet too high in cholesterol for too long." B) "Several factors like my genetics and gender may have contributed to this, but I'm glad that medications can cure it." C) "This explains why my skin was yellow-tinged lately and why I had those pains that spread to my upper back and right shoulder." D) "I suppose the fever and vomiting I had this week was probably a sign of my gallstones too."

C Gallstones can be caused by abnormalities in the composition of bile (increased cholesterol) and stasis of bile. The formation of cholesterol stones is associated with obesity and occurs more frequently in women. These factors cause the liver to excrete more cholesterol into the bile. Estrogen reduces the synthesis of bile acid in women. Cholesterol stones are extremely common in Native Americans. Jaundice and pain that radiates to the upper back and right shoulder are noted signs and symptoms of cholelithiasis. While cholesterol is a key element in the formation of gallstones, the particular amount ingested in the diet is not central to the development of the problem. Surgery, not medication, is the normal treatment modality, and fever and nausea are more closely associated with cholecystitis rather than cholelithiasis.

34-year-old man has been taking up to 2400 mg of ibuprofen per day following a motor vehicle accident several months ago and consequent chronic pain. He has recently been diagnosed with chronic analgesic nephritis as a result of his high analgesic intake. The man is surprised at the diagnosis stating, "I thought that taking too many drugs hurts your liver if anything, not your kidneys." What is the most appropriate response to the man's statement? A) "Your liver does perform most of the detoxification in your body, but your kidneys can perform this role if the liver is unable to." B) "High drug intake can cause your kidneys to be very vulnerable to infections, which is likely what happened in your case." C) "Your kidneys are vulnerable to damage because of how much blood flows through them and the fact that they break down many drugs." D) "It is very rare for someone as young as yourself to have kidney damage like this; usually only older people are vulnerable to kidney damage from drugs."

C High flow and pressure combined with the metabolic transformative of drugs make the kidneys vulnerable to drug toxicity. They do not play a backup role to the liver in the metabolism of drugs, and while structural and functional damage may occur with drug overuse, infection is a less likely consequence. The elderly are particularly vulnerable to drug-related nephritis and nephropathies, but it would be incorrect to conclude that younger people are thus immune.

A toddler brought up in a chaotic, nonnurturing environment may suffer neurological consequences if the parent does not achieve attachment with the child. The nurse knows that which of the following nervous systems listed below is the first to respond to the safety needs of the child? A) Parasympathetic nervous system B) Autonomic nervous system C) Sympathetic nervous system D) Limbic system

C If the ventral vagus is utilized and fails to provide safety, the SNS is recruited first. The other distracters are not the first to respond.

A 25-year-old Asian American man arrives in the emergency room in a panic. Except for a bout with bronchitis a week earlier, he has been healthy his entire life; today he has blood in his urine. What disease has likely caused of his hematuria and how should it be treated? A) Goodpasture syndrome and will be treated with plasmapheresis and immunosuppressive therapy B) Membranous glomerulonephritis and should be treated with corticosteroids C) Immunoglobulin A nephropathy and may be advised to use omega-3 fatty acids to delay progression of disease D) Kimmelstiel-Wilson syndrome and should be treated with medication to control high blood pressure

C Immunoglobulin A (IgA) nephropathy (Buerger disease) is a primary glomerulonephritis characterized by the presence of glomerular IgA immune complex deposits. It can occur at any age, but most commonly occurs with clinical onset in the second and third decades of life. It is more common in males than in females and is the most common cause of glomerular nephritis in Asians. There is no satisfactory treatment for IgA nephropathy. Goodpasture syndrome is a form of glomerulonephritis; treatment includes plasmapheresis to remove circulating anti-GBM antibodies and immunosuppressive therapy to inhibit antibody production. Membranous glomerulonephritis is the most common cause of primary nephrosis in adults, most commonly those in their sixth or seventh decade. It is treated with corticosteroids. In nodular glomerulosclerosis, also known as Kimmelstiel-Wilson syndrome, there is nodular deposition of hyaline in the mesangial portion of the glomerulus. As the sclerotic process progresses in the diffuse and nodular forms of glomerulosclerosis in many cases, early changes in glomerular function can be reversed by careful control of blood glucose levels. Control of high blood pressure and smoking cessation are recommended as primary and secondary prevention strategies in persons with diabetes.

A college student has been experiencing frequent headaches that he describes as throbbing and complaining of difficulty concentrating while studying. Upon cerebral angiography, he is found to have an arteriovenous malformation. Which of the following pathophysiological concepts is likely responsible for his symptoms? A) Increased tissue perfusion at the site of the malformation B) Hydrocephalus and protein in the cerebral spinal fluid C) High pressure and local hemorrhage of the venous system D) Localized ischemia with areas of necrosis noted on CT angiography

C In arteriovenous malformations, a tangle of arteries and veins acts as a bypass between the cerebral arterial and venous circulation, in place of the normal capillary bed. However, the capillaries are necessary to attenuate the high arterial blood pressure before this volume drains to the venous system. As a result, the venous channels experience high pressure, making them to hemorrhage and rupture more likely; the lack of perfusion of surrounding tissue causes neurologic deficits such as learning disorders. Headaches are severe, and people with the disorder may describe them as throbbing (synchronous with their heartbeat). Increased tissue perfusion means that more oxygenated blood is brought to the area, which is not the case. The elevated arterial and venous pressures divert blood away from the surrounding tissue, impairing tissue perfusion. Answer choice B is incorrect since arteriovenous malformation is associated with blood vessels and not the fluid within the ventricles of the brain. Answer choice D is incorrect in that there is blood flow to the area. Ischemia is associated with decreased arterial flow resulting in death to brain tissue.

A 30-year-old male who manages his type 1 diabetes with glyburide presents at the emergency room complaining of headache, confusion, and tachycardia. He has come from a party at which he drank two beers to celebrate running his first half-marathon. Which of the following is likely to be the cause of his complaints? A) Diabetic ketoacidosis B) Hyperosmolar hyperglycemic state C) Hypoglycemia D) Somogyi effect

C In hypoglycemia, headache, difficulty in problem solving, disturbed or altered behavior, coma, and seizures may occur. At the onset, activation of the parasympathetic nervous system often causes hunger, and the initial parasympathetic response is followed by activation of the sympathetic nervous system; this causes anxiety, tachycardia, sweating, and constriction of the skin vessels (i.e., the skin is cool and clammy). In diabetic ketoacidosis, the client typically has a history of 1 or 2 days of polyuria, polydipsia, nausea, vomiting, and marked fatigue. Abdominal pain and tenderness may be experienced without abdominal disease, and the breath has a characteristic fruity smell. The most prominent manifestations of hyperosmolar hyperglycemic state are dehydration and neurologic signs, including grand mal seizures, hemiparesis, Babinski reflexes, aphasia, muscle fasciculations, hyperthermia, hemianopia, nystagmus, and visual hallucinations; the client will also experience excessive thirst. The Somogyi effect describes a cycle of insulin-induced posthypoglycemic hyperglycemic episodes.

An adult male has a new diagnosis of Guillain-Barré syndrome. The nurse knows which of the following pathophysiological processes underlie the deficits that accompany the degeneration of myelin in his peripheral nervous system (PNS)? A) The destruction of myelin causes fewer Schwann cells to be produced in the client's PNS. B) The axonal transport system is compromised by the lack of myelin surrounding nerve cells. C) Unless remyelination occurs, the axon will eventually die. D) A deficit of myelin predisposes the client to infection by potential pathogens.

C In some pathologic conditions, the myelin may degenerate or be destroyed. This leaves a section of the axonal process without myelin while leaving the nearby oligodendroglial or Schwann cells intact. Unless remyelination takes place, the axon eventually dies. A lack of myelin is associated with reduced insulation and impulse conduction. Schwann cells produce myelin, not vice versa, and the myelin is responsible for neither the axonal transport system nor protection against pathogens.

A 24-year-old man is currently in a rehabilitation facility following a spinal cord injury at level T2. He is discussing his long-term options for continence management. Which of the following statements by the client demonstrates he has a clear understanding of the issue? A) "Self-catheterization can limit the recovery of my neural pathways that control my voiding if I do it too often." B) "It's critical that intermittent catheterization be performed using sterile technique." C) "An indwelling catheter certainly would work well, but it comes with a number of risks and possible complications." D) "An indwelling urethral catheter is the option that best minimizes my chance of a urinary tract infection."

C Indwelling catheters carry a risk of infections and kidney stones. Catheterization does not influence the activity of the neural pathways, and intermittent catheterization can be performed using clean technique. Indwelling urethral catheters carry a high risk of urinary tract infections.

A patient is asked to stand with feet together, eyes open, and hands by the sides. Then the patient is asked to close his eyes while the nurse observes for a full minute. What assessment is the nurse performing? A) Segmental reflex B) Posture C) Proprioception D) Crossed-extensor reflex

C Information from the sensory afferents is relayed to the cerebellum and cerebral cortex and is experienced as proprioception or the sense of body movement and position independent of vision. The knee-jerk reflex is a form of stretch reflex. The crossed-extensor reflex serves to integrate motor movements, so they function in a coordinated manner.

When explaining the role of liver Kupffer cells to a group of nursing students, which of the following statements about the function of these cells is most accurate? A) The primary function of Kupffer cells is to secrete bile. B) These cells are the functional unit of the liver and are responsible for all liver secretions. C) The cells are capable of removing and phagocytizing old and defective blood cells. D) The role of the Kupffer cells is to provide at least 50% of cardiac output each minute to each lobular of the liver.

C Kupffer cells are reticuloendothelial cells that are capable of removing and phagocytizing old and defective blood cells, bacteria, and other foreign material from the portal blood as it flows through the sinusoid. This phagocytic action removes enteric bacilli and other harmful substances that filter into the blood from the intestine. Small tubular channels in the liver secrete bile. The functional unit of the liver is lobule. Approximately 25% of blood per minute enters the liver through the hepatic artery.

A 61-year-old woman who has had an upper respiratory infection for several weeks has presented to her family physician with complaints of a recent onset of urinary retention. She reveals to her physician that she has been taking nonprescription cold medications over and above the suggested dose for the past 2 weeks. Which of the following phenomena will her physician most likely suspect is contributing to her urinary retention? A) Cholinergic actions of the cold medicine are triggering internal and external sphincter contraction. B) Antihistamine effects inhibit communication between the pons and the thoracolumbar cord. C) The anticholinergic effects of the medication are impairing normal bladder function. D) Over-the-counter medications such as cold medicine stimulate the parasympathetic nervous system and inhibit bladder emptying.

C Many over-the-counter cold medications have an anticholinergic effect that interferes with normal bladder emptying. These effects on micturition are not a result of cholinergic actions or miscommunication between the pontine micturition center and the spinal cord. Stimulation of the parasympathetic nervous system would tend to increase rather than decrease bladder emptying.

Which of the following clients may be experiencing a sensory focal seizure that has sent an abnormal cortical discharge to the autonomic nervous system (ANS)? A) A 44-year old patient complaining of constant movement and pain in the legs that gets worse when he tries to sleep B) An 85-year-old patient experiencing drooping of the right side of the face and numbness in the right arm and leg C) A 56-year-old complaining of tingling sensations and has both an elevated pulse and BP D) A 22-year-old complaining of a stiff neck and achiness, along with some nausea and vomiting

C Sensory symptoms correlate with the location of seizure activity on the contralateral side of the brain and may involve somatic sensory disturbance (tingling). With abnormal cortical discharge stimulating ANS, tachycardia, diaphoresis, hypo- or hypertension, or papillary changes may be evident. Distracter A is associated with restless legs syndrome (RLS). Distracter B is associated with stroke (CVA). Distracter D is associated with meningitis.

A 24-year-old college student has presented to the campus medical clinic with complaints of frequent, burning urination and has, subsequent to urinalysis, been diagnosed with an acute lower urinary tract infection (UTI) caused by E. coli. What teaching will the clinician most likely provide to the student? A) "This should likely resolve itself if you drink a lot of water and especially cranberry or blueberry juice." B) "Unfortunately, the bacteria causing your infection is no longer responsive to antibiotics, but there are alternative treatments that we can use." C) "Many of these bacteria are now resistant to some antibiotics, but I will take that into account when I choose which antibiotic to prescribe." D) "This likely shows that you have some sort of obstruction in your urinary system, so when that is treated your UTI will likely resolve as well."

C Microbial resistance to TMP-SMX antibiotics is now common in the United States; however, other specific antibiotic options do exist. Cranberry and blueberry juice are more appropriate as preventative rather than curative measures (reduces bacterial adherence to the epithelial lining of the urinary tract), and the majority of uncomplicated UTIs in young women are not the result of urinary obstructions.

A speech therapist is performing a swallowing assessment on a 72-year-old man who has suffered a stroke 3 weeks ago. The man has been NPO (nothing by mouth) since his stroke, and the health care team is considering the introduction of oral food. The speech therapist is cueing the client to swallow to preclude either aspiration of food or pocketing of food in the sides of his mouth. The client most likely to have conscious control over which of the following processes listed below involved in swallowing? A) Initiation of primary peristalsis B) Moving the epiglottis back to cover the larynx C) Moving a bolus to the posterior wall of the pharynx D) Moving the bolus backward in the esophagus

C Moving a bolus to the posterior wall of the pharynx is a component of the oral, or voluntary, phase of swallowing. Initiation of primary peristalsis, moving the epiglottis back to cover the larynx, and moving the bolus backward in the esophagus are all involuntary components of the pharyngeal and esophageal phases of swallowing.

The parents of a 3-year-old boy have brought him to a pediatrician for assessment of the boy's late ambulation and frequent falls. Subsequent muscle biopsy has confirmed a diagnosis of Duchenne muscular dystrophy. Which of the following teaching points should the physician include when explaining the child's diagnosis to his parents? A) "Your son's muscular dystrophy is a result of faulty connections between muscles and the nerves that normally control them." B) "He'll require intensive physical therapy as he grows up, and there's a good chance that he will outgrow this problem as he develops." C) "Your son will be prone to heart problems and decreased lung function because of this." D) "His muscles will weaken and will visibly decrease in size relative to his body size throughout his childhood."

C Muscular dystrophy is associated with cardiac and respiratory complications. It does not involve the nervous system, and the problem will not dissipate with time. While muscles become weakened, pseudohypertrophy means that their size does not decrease.

A 35-year-old female ultramarathon runner is admitted to the hospital following a day-long, 50-mile race because her urinary volume is drastically decreased and her urine is dark red. Tests indicate that she is in the initiating phase of acute tubular necrosis. Why is her urine red? A) Hematuria B) Hemoglobinuria C) Myoglobinuria D) Kidney bleeding

C Myoglobinuria, which can cause acute tubular necrosis via intratubular obstruction, involves the leaching of myoglobin from skeletal muscle into the urine, bypassing the usual filtration by the glomerulus. Excess exercise and muscle trauma can contribute to this. While both hemoglobinuria and myoglobinuria discolor the urine, hemoglobinuria results from hemolysis following a reaction to a blood transfusion, whereas myoglobinuria involves muscle damage.

A 15-year-old boy with type 1 diabetes is exasperated by his regimen of blood sugar monitoring and insulin administration and has told his mother that he wants to scale both back. Which of the following responses by his mother is most accurate? A) "I know it's not fun, but you're even more susceptible to complications when you're young if you don't stay on top of your diabetes." B) "Even though you might save some time and energy by doing this, remember that high blood sugars cause a lot of pain and will cause you to gain weight." C) "Even though it's hard to do, you need to continue, so you don't go blind or need a kidney transplant down the road." D) "You need to be vigilant now if you want to be free of diabetes when you grow up."

C Nephropathies and retinopathies are common complications of poor glycemic control. Complications of diabetes are not noted to be more acute in younger individuals, and hyperglycemia is not normally associated with pain or weight gain. Conscientious diabetes control does not result in a resolution of the disease.

A patient with distal symmetric polyneuropathy usually begins to complain of A) burning in the big toe. B) painful cramps in the calves. C) loss of feeling or touch in the feet. D) seeping wounds in lower legs.

C Peripheral neuropathy is often associated with the insensate foot. The loss of feeling, touch, sensation, and position sense increases the risk of falling, serious burns, and injuries to the feet. Burning in the big toe is usually associated with gout. Cramps in the legs may be electrolyte imbalances, and seeping wounds in lower legs could be a result of obesity, lymphedema, or diabetes to name a few.

The nurse assessing a renal failure patient for encephalopathy caused by high uremic levels may observe which of the following clinical manifestations? A) Severe chest pain with pericardial friction rub on auscultation B) Stiff immobile joints and contractures C) Loss of recent memory and inattention D) Pruritus with yellow hue to skin tone

C Reductions in alertness and awareness are the earliest and most significant indications of uremic encephalopathy. These often are followed by an inability to fix attention, loss of recent memory, and perceptual errors in identifying people and objects. Answer choice A refers to pericarditis; answer choice B refers to arthritis; answer choice D refers to liver involvement.

One of the most reliable predictors for worsening autosomal dominant polycystic kidney disease is A) serum creatinine levels. B) blood urea nitrogen (BUN) level. C) urine albumin excretion (UAE). D) urine specific gravity.

C Serum creatinine levels have not been found to be an effective predictor marker for worsening ADPKD, but urine albumin excretion (UAE) has been determined a reliable predictor, as have increased electrolytes and hematuria.

A 70-year-old male with a 40 pack-year history of smoking and long-standing non-insulin-dependent diabetes has been diagnosed with pancreatic cancer. Which of the following teaching points should the physician provide? A) "While this is indeed serious, you should know that you have a good chance of beating this disease with appropriate treatment." B) "Most likely your pattern of high alcohol intake over the years contributed to your cancer." C) "You will likely be facing surgery in the near future, but know that this is very unlikely to eliminate your cancer." D) "I know it may seem trivial at this point, but the levels of pain that accompany cancer of the pancreas are normally quite low."

C Surgery is the most common treatment modality for pancreatic cancer, but it does not commonly result in the resolution of the disease. Pancreatic cancer has a very poor prognosis, and alcohol is not a common contributor. Pancreatic cancer is noted to be exceptionally painful.

A 63-year-old woman has visited a physician because she has been intermittently passing blood-tinged urine over the last several weeks, and cytology has confirmed a diagnosis of invasive bladder cancer. Which of the following statements by the physician is most accurate? A) "There are new and highly effective chemotherapy regimens that we will investigate." B) "Fortunately, bladder cancer has a very low mortality rate, and successful treatment is nearly always possible." C) "It's likely that you'll need surgery, possibly a procedure called a cystectomy." D) "Unfortunately, there are nearly no treatment options for this type of cancer, but we will focus on addressing your symptoms."

C Surgical interventions are common in the treatment of bladder cancer. Effective chemotherapeutic regimens are not yet available, though there are certainly treatment options. The mortality rate of bladder cancer is high, at around 25%.

A nurse at a long-term care facility provides care for an 85-year-old man who has had recent transient ischemic attacks (TIAs). Which of the following statements best identifies future complications associated with TIAs? TIAs A) are an accumulation of small deficits that may eventually equal the effects of a full CVA. B) are a relatively benign sign that necessitates monitoring but not treatment. C) resolve rapidly but may place the client at an increased risk for stroke. D) are caused by small bleeds that can be a warning sign of an impending stroke.

C TIAs can be considered a warning sign for future strokes. They are not hemorrhagic in nature, and their effects are not normally cumulative. They may require treatment medically or surgically.

After being thrown off the back of a bull, the bull rider can move his arms but has loss of motor function in the lumbar and sacral segments of the spinal cord. This is usually referred to as A) tetraplegia. B) quadriplegia. C) paraplegia. D) anterior cord syndrome.

C Tetraplegia and quadriplegia are loss of motor or sensory function after damage to neural structures in the cervical segments of the spinal cord. Paraplegia refers to loss of motor or sensory function in thoracic, lumbar, or sacral segments. The arms function as normal. Anterior cord syndrome includes loss of motor function provided by the corticospinal tracts and loss of pain and temperature sensation from damage to the lateral spinothalamic tracts.

When explaining to a class of nursing students enrolled in pathophysiology, the instructor states, "the majority of energy used by the kidney is for A) filtration of drugs out of the body." B) secretion of erythropoietin for production of RBCs." C) active sodium transport mechanisms." D) removal of excess glucose from the blood."

C The bulk of energy used by the kidney is for active sodium transport mechanisms that facilitate sodium reabsorption and cotransport of other electrolytes and substances such as glucose and amino acids.

When explaining about the passage of urine to a group of nursing students, the clinic nurse asks them which muscle is primarily responsible for micturition? Their correct reply is the A) urinary vesicle. B) trigone. C) detrusor. D) external sphincter.

C The detrusor muscle is the muscle of micturition. When it contracts, urine is expelled from the bladder. External sphincter is a circular muscle that surrounds the urethra distal to the base of the bladder and can stop micturition when it is occurring. Trigone is a smooth triangular area that is bounded by the ureters and the urethra. Urinary vesicle is another name for the bladder.

A 71-year-old male has been recently diagnosed with a stage III tumor of colorectal cancer and is attempting to increase his knowledge base of his diagnosis. Which of the following statements about colorectal cancer demonstrates a sound understanding of the disease? A) "If accurate screening test for this type of cancer existed, it could likely have been caught earlier." B) "The NSAIDs and aspirin that I've been taking for many years probably contributing to me getting cancer." C) "While diet is thought to play a role in the development of colorectal cancer, the ultimate causes are largely unknown." D) "A large majority of patients who have my type of colon cancer survive to live many more years."

C The etiology of cancer of the colon and rectum remains largely unidentified, though dietary factors are thought to exist. The prognosis, especially with stage III tumors, is poor. Simple and accurate screening tests do exist for colorectal cancer, while drugs are not implicated in the etiology.

A 9-year-old girl has a diffuse collection of symptoms that are indicative of deficits in endocrine and autonomic nervous system control. She also suffers from persistent fluid and electrolyte imbalances. The nurse knows which of the following aspects of the nervous system listed below would her health care providers focus their diagnostic efforts on? A) Her afferent and efferent cranial nerve function B) Possible damage to her pons and medulla C) Impaired function of her hypothalamus D) Potential damage to the girl's cerebellum

C The hypothalamus plays a central role in the maintenance of fluid and electrolyte balance and in the maintenance of endocrine control. Various cranial nerves, the hindbrain, and the cerebellum would be less likely to be implicated.

A male infant who is 48 hours postpartum is undergoing phototherapy for the treatment of jaundice and accompanying high levels of bilirubin. Place the following components of the production of bilirubin in the correct chronological order. Use all the options. A) Conjugated bilirubin B) Urobilinogen C) Red blood cells D) Biliverdin E) Free bilirubin

C D E A B Hemoglobin from the red blood cell is broken down to form biliverdin, which is rapidly converted to free bilirubin. Inside the hepatocytes, free bilirubin is converted to conjugated bilirubin. Conjugated bilirubin is secreted as a constituent of bile, and in this form, it passes through the bile ducts into the small intestine. In the intestine, approximately one half of the bilirubin is converted into a highly soluble substance called urobilinogen by the intestinal flora.

A physician who is providing care for a 71-year-old male client with a recent diagnosis of renal failure and an acid-base imbalance is explaining some of the underlying etiology of the man's diagnoses to him and his family. Which of the following phenomena would most accurately underlie the teaching that the physician provides? A) The kidneys are integral to the reabsorption of hydrogen ions and maintenance of a low pH. B) Blood buffer systems and respiratory control can compensate for inadequate renal control of pH. C) The kidneys have the primary responsibility for eliminating excess hydrogen ions from the body. D) pH is kept at an optimal level through the renal secretion of bicarbonate ions in blood filtrate.

C The kidneys have the primary responsibility for maintaining body pH by eliminating excess hydrogen ions from the body, a function that blood buffer systems and respiratory control are incapable of. Hydrogen ions must be eliminated, not retained, and bicarbonate must be produced as part of buffer action, not secreted.

A 47-year-old woman was diagnosed with amyotrophic lateral sclerosis 3 years ago and has experienced a progressive onset and severity of complications. She has been admitted to a palliative care unit due to her poor prognosis? What assessments and interventions should the nursing staff of the unit prioritize in their care? A) Assessment and documentation of cognitive changes, including confusion and restlessness B) Regular pain assessment and administration of opioid analgesics as needed C) Assessment of swallowing ability and respiratory status D) Cardiac monitoring and administration of inotropic medications

C The late stages of ALS normally involve deterioration in swallowing and speech and in the respiratory musculature. Cognitive changes are not common complications, and pain and cardiac complications are not noted to be paramount in the course of ALS.

A 51-year-old male has been diagnosed with alcohol-induced liver disease. He admits to the nurse providing his care that, "I know what the lungs do, and I know what the heart does, but honestly I have no idea what the liver does in the body." Which of the following statements would best underlie the explanation that the nurse provides? A) The liver is responsible for the absorption of most dietary nutrients as well as the production of growth hormones. B) The liver contributes to the metabolism of ingested food and provides the fluids that the GI tract requires. C) The liver metabolizes most components of food and also cleans the blood of bacteria and drugs. D) The liver maintains a balanced level of electrolytes and pH in the body and stores glucose, minerals, and vitamins.

C The liver metabolizes protein, carbohydrate, and fat. As well, it metabolizes drugs and removes bacteria by Kupffer cells. Absorption of nutrients takes place in the intestines, and the liver does not produce the bulk of fluids secreted in the GI tract. The liver does not have a primary role in the maintenance of acid-base or electrolyte balance.

A patient who has suffered a spinal cord injury at C4 is experiencing a sudden change in condition. His BP is 186/101; heart rate is 45; and he is profusely sweating and complaining of "not feeling right." The nurse should A) call a "Code Blue." B) page physician stat. and ask for an antihypertensive medication. C) palpate his bladder for overdistention. D) place his bed flat and elevate the foot of the bed.

C The most common causes of spastic bladder dysfunction are spinal cord lesions such as spinal cord injury, herniated intervertebral disk, vascular lesions, tumors, and myelitis. Because the injury interrupts CNS control of sympathetic reflexes in the spinal cord, severe hypertension, bradycardia, and sweating can be triggered by insertion of a catheter or mild overdistention of the bladder. The patient does not qualify for a Code Blue since he still has a pulse and is breathing. Antihypertensive medication is not necessary if the bladder is emptied. Placing him flat with the foot of the bed elevated will not help this situation.

A 62-year-old woman with high blood pressure is to begin long-term treatment with a thiazide diuretic that she thinks she will need to take for some time. What should the nurse expect to happen to her potassium and calcium levels? A) Her potassium and calcium levels will not change. B) Her potassium and calcium levels will both go down. C) Her potassium level will drop, but her calcium level may rise. D) Her potassium level will rise, but her calcium level may drop.

C Thiazide diuretics increase the loss of potassium in urine. Because calcium is actively reabsorbed in the distal convoluted tubule, it is likely that her calcium level will go up, especially if she takes it for a long time.

A patient in the ICU has been diagnosed with hypovolemic shock. His BP is 88/53, heart rate 122, and respiratory rate 26. Given these vital signs, the nurse should expect the urine output to be A) maintained between 30 and 50 mL/hour with no sediment in the bag. B) increased to 60+ mL/hour with dilute urine. C) decreased below 30 mL/hour with decreased GFR. D) the patient's normal amount with dark, concentrated urine.

C Under conditions of decreased perfusion or increased sympathetic nervous system stimulation, blood flow is redistributed away from the cortex toward the medulla. This redistribution of blood flow decreased glomerular filtration while maintaining the urine-concentrating ability of the kidneys, a factor that is important during conditions such as shock. The urine output would not increase, nor be normal.

A nurse educator is performing client education with a 51-year-old man who has been recently diagnosed with chronic kidney disease. Which of the following statements by the client would the nurse most likely want to correct or clarify? A) "I'll be prone to anemia, since I'm not producing as much of the hormone that causes my bones to produce red blood cells." B) "My heart rate might go up because of my kidney disease, and my blood might be a lot thinner than it should be." C) "My kidney problems increase my chance of developing high blood pressure or diabetes." D) "I'll have a risk of either bleeding too easily or possibly clotting too quickly, though dialysis can help minimize these effects."

C While high blood pressure can be causative of, or consequent to, renal failure, diabetes is not normally a result of existing CKD. Persons with renal failure are indeed prone to anemia, increased heart rate, decreased blood viscosity, and coagulopathies. The risk of bleeding and thrombotic disorders can be partially mitigated by dialysis.

A 30-year-old man with a diagnosis of type 1 diabetes is aware of the multiple effects that insulin has on his metabolism. Which of the following physiological processes are actions of insulin? Select all that apply. A) Increasing the metabolic needs of body cells B) Promoting the breakdown of stored triglycerides C) Facilitating triglyceride synthesis from glucose in fat cells D) Inhibiting protein breakdown E) Promoting glucose uptake by target cells

C, D, E The actions of insulin are threefold: (1) it promotes glucose uptake by target cells and provides for glucose storage as glycogen; (2) it prevents fat and glycogen breakdown; and (3) it inhibits gluconeogenesis and increases protein synthesis. Insulin acts to promote fat storage by increasing the transport of glucose into fat cells. It also facilitates triglyceride synthesis from glucose in fat cells and inhibits the intracellular breakdown of stored triglycerides. Insulin also inhibits protein breakdown. It does not directly influence the metabolic needs of body cells.

Following a car accident of a male teenager who did not have his seatbelt on, he arrived in the emergency department with a traumatic brain injury. He has severe cerebral edema following emergent craniotomy. Throughout the night, the nurse has been monitoring and reporting changes in his assessment. Which of the following assessments correspond to a supratentorial herniation that has progressed to include midbrain involvement? Select all that apply. A) Clouding of consciousness B) Decorticate posturing with painful stimulation C) Pupils fixed at approximately 5 mm in diameter D) Respiration rate of 40 breaths/minute E) Decerebrate posturing following painful stimulation of the sternum

C, D, E With midbrain involvement, pupils are fixed and midsized (5 mm in diameter), and reflex adduction of the eyes is impaired; pain elicits decerebrate posturing; and respirations change from Cheyne-Stokes respiration to neurogenic hyperventilation. Cloudiness of consciousness occurs in early diencephalic stages. Decorticate posturing with pain occurs in the diencephalic stage.

A middle-aged man with diabetes reports that he must strain to urinate and that his urine stream is weak and dribbling. He also reports feeling that his bladder never really empties. The nurse knows that all of his complaints are likely caused by which of the following medical diagnoses? A) Detrusor muscle areflexia B) Detrusor-sphincter dyssynergia C) Uninhibited neurogenic bladder D) Bladder atony with dysfunction

D Diabetes causes peripheral neuropathy, which can affect the sensory axons of the urinary bladder. Bladder atony with dysfunction is a frequent complication of diabetes mellitus.

A nursing student having trouble moving his head from side to side is likely experiencing a problem with which type of neurons? A) General visceral efferent neurons B) Preganglionic neurons C) Parasympathetic postganglionic neurons D) Pharyngeal efferent neurons

D Pharyngeal efferent neurons innervate brachial arch skeletal muscles, muscles of mastication and facial expression, and muscles of the pharynx and larynx. They also innervate muscles responsible for moving the head. General visceral efferent neurons and preganglionic neurons (same) innervate smooth and cardiac muscle and glandular cells of the body, most of which are in the viscera. Parasympathetic postganglionic neurons have no effect in moving the head.

Not realizing that its surface was hot, a woman has quickly withdrawn her hand from the surface of a bowl that she was removing from a microwave. Which of the following phenomena has facilitated the rapid movement of her hand in response to the painful stimulus? A) Her midbrain has rapidly responded to the nociceptive stimuli and induced arm flexion. B) The withdrawal reflex of her peripheral nervous system has quickly mediated between afferent and effector neurons. C) The forebrain has mediated a protective spinal cord reflex. D) Her CNS has enacted a protective response received by neurons that innervate her arm muscles.

D A reflex is a CNS-mediated response to a painful stimulus that involves an afferent neuron and an effector neuron. The midbrain and forebrain do not necessarily participate in the sensory or motor components of the response.

A patient has just been diagnosed with acute glomerulonephritis. Which question should the nurse ask this client in attempting to establish a cause? A) "Do you have a history of heart failure?" B) "Have you recently had kidney stones?" C) "Have you ever been diagnosed with diabetes?" D) "Have you had any type of infection within the last 2 weeks?"

D Acute postinfectious glomerulonephritis usually occurs after infection with certain strains of group A β-hemolytic streptococci and is caused by deposition of immune complexes of antibody and bacterial antigens. Other organisms can also cause this infection.

A patient asks the nurse what it means when the doctor said that he had adenocarcinoma of the bladder. Reviewing the pathophysiologic principles behind this type of cancer, the nurse knows A) it is a low-grade tumor that is readily cured with bladder surgery. B) after resection of the cancer, the prognosis is excellent with this type of cancer cell. C) that these types of cancer cells are very invasive to the tissue; therefore, the entire bladder must be removed. D) this is a rare but highly metastatic tumor that has a very poor prognosis.

D Adenocarcinoma is rare and highly metastatic. Answer choices A and B relate to urothelial carcinoma; answer choice C relates to squamous cell carcinoma.

While on tour, a 32-year-old male musician has presented to the emergency department of a hospital after a concert complaining of severe and sudden abdominal pain. He admits to a history of copious alcohol use in recent years, and his vital signs include temperature 46.8°C (101.8°F), blood pressure 89/48 mm Hg, and heart rate 116 beats/minute. Blood work indicates that his serum levels of C-reactive protein, amylase, and lipase are all elevated. Which of the following diagnoses would the care team suspect first? A) Hepatitis C B) Cholecystitis C) Liver cirrhosis D) Acute pancreatitis

D Alcohol use, fever, hypotension, and tachycardia are often associated with pancreatitis, as are elevated serum amylase and lipase levels. These enzymes would unlikely rise in cases of hepatitis, cholecystitis, or cirrhosis. The precise mechanisms whereby alcohol exerts its action are largely unknown. The capacity for oxidative and nonoxidative metabolism of ethanol by the pancreas and the harmful by-products that result have been related to the disease process. Hepatitis C has an incubation period. Most adults who acquire the infection usually are asymptomatic. Jaundice is uncommon. Direct measurement of HCV in the serum remains the most accurate test for infection. Cirrhosis represents the end stage of chronic liver disease. The end result is liver failure that affects many organs. The patients usually have anemia, thrombocytopenia, endocrine disorders, skin lesions, azotemia and renal failure, and hepatic encephalopathy.

A 70-year-old male has been diagnosed with a stroke that resulted in an infarct to his cerebellum. Which of the following clinical findings would be most closely associated with cerebellar insult? A) Flaccid loss of muscle tone B) Difficulty in starting movement, stopping movement, and maintaining rhythmic movements. C) Tremor, rigidity, and bradykinesia D) Unsteady gait and difficulty in speaking and swallowing

D An unsteady gait characterizes cerebellar ataxia, and both swallowing and speaking are partly the domain of the cerebellum. Flaccid loss of muscle tone is not noted to accompany cerebellar insult, and difficulties with starting movement, stopping movement, and maintaining rhythmic movements are indicative of basal ganglia disorders. Tremor, rigidity, and bradykinesia are associated with Parkinson disease

Which of the following statements best describes an aspect of the normal process of glucose metabolism? A) Blood glucose levels are primarily a result of the timing, quantity, and character of food intake. B) Ingested glucose that is not needed for cellular metabolism circulates in the blood until it is taken up to meet cellular needs. C) Blood glucose levels are kept in a steady state by selective excretion and reuptake by the kidneys. D) Glucose that exceeds metabolic needs is converted and stored by the liver.

D Approximately two thirds of the glucose that is ingested with a meal is removed from the blood and stored in the liver as glycogen. Between meals, the liver releases glucose as a means of maintaining blood glucose within its normal range. Normal glucose metabolism does not involve large variations in blood glucose levels in response to food intake, and excess glucose does not normally remain in circulation. Glucose levels are not primarily maintained by the kidneys.

A diabetes education nurse is teaching a group of recently diagnosed diabetics about the potential genitourinary complications of diabetes and the consequent importance of vigilant blood glucose control. Which of the following teaching points best conveys an aspect of bladder dysfunction and diabetes mellitus? A) "People with diabetes are highly susceptible to urethral obstructions, and these can heal more slowly and cause more damage than in people without diabetes." B) "High blood sugar results in a high glucose level in your urine, and this can make your bladder muscle less able to fully empty the bladder." C) "Many people with diabetes find it necessary to live with an indwelling catheter to ensure their bladders do not become too full." D) "It's important for you to empty your bladder frequently because diabetes carries risks of kidney damage that can be exacerbated by incomplete bladder emptying."

D Diabetics are vulnerable to peripheral neuropathies that can be somewhat mitigated by regular voiding; they are also especially vulnerable to renal damage from high blood sugars, a situation that is worsened when accompanied by incomplete bladder emptying. Urethral obstructions are not a noted complication of diabetes, and indwelling catheter placement is not normally necessary. High blood sugars do not necessarily yield high-glucose urine, and the bladder deficits associated with diabetes are neurological in nature rather than a result of particular urine chemistry.

A 42-year-old male has been diagnosed with renal failure secondary to diabetes mellitus and is scheduled to begin dialysis soon. Which of the following statements by the client reflects an accurate understanding of the process of hemodialysis? A) "It's stressful knowing that committing to dialysis means I can't qualify for a kidney transplant." B) "I know I'll have to go to a hospital or dialysis center for treatment." C) "Changing my schedule to accommodate 3 or 4 hours of hemodialysis each day will be difficult." D) "I won't be able to go about my normal routine during treatment."

D Hemodialysis requires the client to remain connected to dialysis machinery, whereas peritoneal dialysis allows for activity during treatment. Dialysis does not disqualify an individual from receiving a transplant. Dialysis does not require attendance at a dialysis center, and patients can be taught to perform the dialysis in their home with a family member in attendance. Hemodialysis is normally conducted three times weekly, not once per day.

A client with a diagnosis of depression has been prescribed a medication that ultimately increases the levels of the neurotransmitter serotonin between neurons. Which of the following processes will accompany the actions of the neurotransmitter in a chemical synapse? A) Two-way communication between neurons is permitted in contrast to the one-way communication in electrical synapses. B) Communication between a neuron and the single neuron it is connected with will be facilitated. C) The neurotransmitter will cross gap junctions more readily. D) More neurotransmitters will cross the synaptic cleft and bond with postsynaptic receptors.

D In chemical synapses, neurotransmitters cross the synaptic cleft and bond with postsynaptic receptors to facilitate communication between neurons. This communication is one way, not two way, and each neuron has synaptic connections with thousands of other neurons. Gap junctions are associated with electrical synapses, not chemical synapses.

A 79-year-old male resident of a long-term care facility has contracted Clostridium difficile and is experiencing consequent diarrhea. Auscultation of the man's abdomen indicates hyperactive bowel sounds. What process in the man's small intestine is most likely accompanying his current status? A) Pathogenic microorganisms are causing dilation of his small intestine, increasing motility. B) Segmentation waves have become more frequent as a result of his infection. C) Intestinal stasis brought on by infection is preventing his small intestine from sufficiently slowing the rate of motility. D) Inflammation is accompanied by an increase in peristaltic movements of his small intestine.

D Inflammation of the small intestine is accompanied by an increase in motility, an effect that is the result of increased peristaltic waves. Segmentation waves are responsible primarily for mixing rather than moving food. Neither dilation nor inadequate slowing of passage contents is responsible for the increase in motility.

While reviewing the role of glucagon in regard to regulation of blood glucose, the nurse knows which of the following situations could lead to an inhibition of glucagon release? A) A sharp decrease in blood glucose concentration B) Recent strenuous physical activity C) Recent intake of large amounts of protein-rich food. D) An increase in glucose levels.

D Low blood sugar, intake of protein, and strenuous physical activity are associated with glucagon release. Lowered cellular metabolic needs and/or increased glucose levels would inhibit glucagon release.

A 77-year-old woman has been admitted to hospital following several weeks of increasing fatigue. On observation, she is pale, and blood work indicates she has low hemoglobin and red cell counts. Stool tests for occult blood are positive, and following endoscopy, she has been diagnosed with an upper GI bleed that has been shown to originate in her stomach. She admits to regularly exceeding the recommended doses of nonsteroidal anti-inflammatory drugs (NSAIDs) in an effort to control her rheumatoid arthritis. Which of the following phenomena is most likely responsible for her present health problems? A) NSAIDs increase the gastric production of gastrin, increasing gastric secretions and lowering stomach pH. B) Drugs such as NSAIDs increase the H+ levels and thus decrease gastric pH, resulting in insult to the stomach lining. C) NSAIDs, aspirin, and other drugs increase prostaglandin synthesis, resulting in disruption of cellular structures lining the stomach. D) NSAIDs can disrupt the permeability of the gastric mucosa, causing hydrogen ions to accumulate in the mucosal cells of the lining.

D NSAIDs damage the mucosal barrier, allowing hydrogen ions to damage cells of the stomach lining. They do not directly increase gastrin production or H+ levels, and decreased, not increased, prostaglandin synthesis would potentially compromise the gastric surfaces.

A 46-year-old man who is obese has received news that he has type 2 diabetes. He is in the process of determining a plan of care with an interdisciplinary team at a hospital-based diabetes clinic. The nurse knows that the most likely treatment plan for the man will include A) injectable insulin and nutrition management. B) weight loss, glucose monitoring, and lifestyle modification. C) continuous subcutaneous insulin infusion (CSII) and nutrition management. D) oral antihyperglycemic medications and weight loss measures.

D Persons with type 2 diabetes would unlikely require insulin initially, and oral medications are likely to be of benefit as an addition to lifestyle modifications and weight loss.

After several months of persistent heartburn, a 57-year-old female client has been diagnosed with gastroesophageal reflux disease (GERD). Which of the following treatment regimens is likely to best address the woman's health problem? A) Surgical correction of the incompetent pylorus B) Antacids; avoiding positions that exacerbate reflux; a soft-textured diet C) Weight loss and administration of calcium channel blocking medications D) Proton pump inhibitors; avoiding large meals; remaining upright after meals

D Proton pump inhibitors block the final stage of gastric acid production, effectively controlling the root cause of the esophageal damage associated with GERD. The pylorus is not involved, and a soft diet is not indicated. Calcium channel-blocking drugs would not address the problem. Calcium channel blockers are primarily heart disease drugs that relax blood vessels and increase the supply of blood and oxygen to the heart while also reducing the heart's workload.

A 48-year-old woman has been diagnosed with extrahepatic cholestasis following a thorough history, ultrasound, and blood work. Which of the following symptoms most likely caused her to seek medical treatment, and what consequence to her health problem would the medical team anticipate? A) Complaints of lower flank pain with consequences of impaired fat metabolism B) Anorexia with consequences of impaired drug metabolism and blood filtration C) Skin xanthomas (focal accumulations of cholesterol) with consequences of increased risk of bleeding due to deficient clotting factors D) Pruritus with consequences of deficient levels of fat-soluble vitamins

D Pruritus is the most common symptom of cholestasis, and deficiencies in fat-soluble vitamins such as A, D, and K are frequent. Flank pain is not a noted complaint, and bile does not contribute to drug metabolism, blood filtration, or clotting factor production.

The nurse knows that which of the following treatment plans listed below is most likely to be prescribed after a computed tomography (CT) scan of the head reveals a new-onset aneurysmal subarachnoid hemorrhage? A) Stat administration of tissue plasminogen activator (tPA) B) Administration of a diuretic such as mannitol to reduce cerebral edema and ICP C) Monitoring in the ICU for signs and symptoms of cerebral insult D) Craniotomy and clipping of the affected vessel

D Surgery for treatment of aneurysmal subarachnoid hemorrhage involves craniotomy and inserting a specially designed silver clip that is tightened around the neck of the aneurysm. Administration of tPA would exacerbate bleeding, and a diuretic would not address the issue of bleeding. Monitoring alone would be an insufficient response given the severity of the problem.

A 20-year-old woman has visited her family physician due to occasional bouts of bloody diarrhea over the past several weeks, a phenomenon that she experienced 2 years prior as well. Her physician has diagnosed her with ulcerative colitis based on her history and visualization of the affected region by colonoscopy and sigmoidoscopy. Which of the following pathophysiological phenomena is most likely to underlie the client's health problem? A) Fissures and crevices developing in the mucosa that are seen as a characteristic "cobblestone" appearance B) Erosion of the endothelial lining of the distal small intestine by a combination of genetic, autoimmune, and environmental factors C) Compromise of the mucosal layer of the large intestinal surface by the effects of H. pylori D) Mucosal hemorrhages that have developed into crypt abscesses, which have in turn necrotized and ulcerated

D The etiology and course of ulcerative colitis involves mucosal hemorrhages developing into crypt abscesses, with consequent necrosis and ulceration. "Cobblestone" appearance of intestinal mucosa is associated with Crohn disease. Ulcerative colitis is confined to the colon and rectum, and H. pylori is not commonly implicated in the etiology.

A 60-year-old male office worker presents to a clinic complaining of new onset of lower back pain that has been worsening over the last 6 weeks. The nurse knows which of the following components of his physical assessment and history is most indicative of a serious pathological process (like aortic aneurysm or cancer)? A) His pain is relieved by extended bed rest. B) When supine, passive rising of his leg to 90 degrees results in hamstring pain. C) He has needed regular nonsteroidal anti-inflammatory drugs to control the pain in recent weeks. D) His onset of pain has been gradual, and he has no prior history of lower back problems.

D The gradual onset of back pain unrelated to injury and initial presentation after age 50 are considered red flags for more serious pathologies such as aortic aneurysm, malignancy, or compression fracture. Pain that is aggravated by lying down is a red flag for malignancy or infection. The onset of hamstring pain at 90 degrees of hip flexion is a normal finding. The need for and use of NSAIDs for lower back pain relief are not indicative of a serious pathology in and of itself.

A 51-year-old male professional is in the habit of consuming six to eight rum and cokes each evening after work. He assures the nurse practitioner who is performing his regular physical exam that his drinking is under control and does not have negative implications for his work or family life. How could the nurse best respond to the client's statement? A) "You are more than likely inflicting damage on your liver, but this damage would cease as soon as you quit drinking." B) "That may be the case, but you are still creating a high risk of hepatitis A or B or liver cancer." C) "In spite of that, the amount of alcohol you are drinking is likely to result first in cirrhosis and, if you continue, in hepatitis or fatty liver changes." D) "When your body has to regularly break down that much alcohol, your blood and the functional cells in your liver accumulate a lot of potentially damaging toxic byproducts."

D The hepatic effects of alcohol use are related to the accumulation of toxic metabolites in the hepatocytes and blood. Damage can continue even after an individual stops drinking. Specific consequences do not usually include HAV, HBV, or liver cancer. Cirrhosis represents the culmination, not the beginning, of negative hepatic effects.

A patient in a hospital is frustrated at the inconvenience of having to collect his urine for an entire day and night as part of an ordered 24-hour urine collection test. He asks the nurse why the test is necessary since he provided a single urine sample 2 days ago. How could the nurse best respond to the patient's question? A) "A single urine sample lets your care team determine if there are bacteria in your urine, but other tests of urine chemistry need a longer-term view." B) "Current lab tests aren't able to detect the small quantities of most substances contained in a single urine sample. C) "Only a longer-term test is able to show whether your kidneys are letting sugar spill out into your urine." D) "Often why an abnormal substance shows up in urine test, a 24-hour urine collection is needed to determine exactly how much it is present in your urine."

D Twenty-four-hour urine tests are often used to quantify the amount of substances, such as proteins, that an individual's kidneys are spilling. Single urine samples are able to assess more parameters than just the presence of bacteria, and they are sufficient in quantity to detect numerous substances such as glucose.

A patient in the intensive care unit who has a brain tumor has experienced a sharp decline. The care team suspects that water and protein have crossed the blood-brain barrier and been transferred from the vascular space into the client's interstitial space. Which of the following diagnoses best captures this pathophysiology? A) Focal hypoxia B) Cytotoxic edema C) Hydrocephalus D) Vasogenic edema

D Vasogenic edema occurs with conditions that impair the function of the blood-brain barrier and allow transfer of water and protein from the vascular into the interstitial space. It occurs in conditions such as tumors, prolonged ischemia, hemorrhage, brain injury, and infectious processes. Focal hypoxia is associated with localized delivery of blood with inadequate oxygen, and cytotoxic edema is an absolute increase in intracellular fluid. Hydrocephalus is an abnormal increase in CSF volume in any part or all of the ventricular system.

During a late night study session, a pathophysiology student reaches out to turn the page of her textbook. Which of the following components of her nervous system contains the highest level of control of her arm and hand action? A) Cerebellum B) Thalamus C) Basal ganglia D) Frontal lobe

D While intentional movement involves input from various components of the nervous system including the cerebellum, thalamus, and basal ganglia, primary control and coordination are controlled by the motor cortex in the frontal lobe.

Which of the following individuals is at the highest risk of developing a urinary tract infection (UTI)? A) A 60-year-old man with a history of cardiovascular disease who is recovering in hospital from a coronary artery bypass graft B) A 66-year-old man undergoing dialysis for the treatment of chronic renal failure secondary to hypertension C) A 38-year-old man with high urine output due to antidiuretic hormone insufficiency D) A 30-year-old woman with poorly controlled diabetes mellitus

D Young women as well as persons with diabetes are at high risk of UTIs. Neither postsurgical recovery nor renal failure is necessarily a direct risk for UTI development, and high urine output would prevent decrease rather than increase in UTI risk.

Place the following components of the gastrointestinal tract in the chronological order that a bolus of food would pass through them. Use all the options. A) Ileum B) Pylorus C) Jejunum D) Hiatus E) Cecum F) Duodenum

D B F C A E Ingested food and fluids enter the stomach through the hiatus, exit through the pylorus, and pass through the three subdivisions of the small intestine: the duodenum, jejunum, and ileum. The cecum is a component of the large bowel.


Ensembles d'études connexes

Introduction for Nutrition Chapter 2

View Set

ENTREP: entrepreneur, entrepreneurship, entrepreneurialism

View Set

Give Me Liberty Chapter 15: What is Freedom? Reconstruction 1865-1877

View Set

Unit 12: Implementation and Presentation

View Set

Management 101 Belmont University Loes CH 1

View Set

Benchmark oraciones and definitions

View Set

PSYC 150 Drug and alcohol abuse Chapter 1

View Set

Intro to nervous system exam review

View Set